gastrointestinal/ nutrition

¡Supera tus tareas y exámenes ahora con Quizwiz!

A client diagnosed with cirrhosis is experiencing pruritus. Which strategies are appropriate for the nurse to teach the client to promote comfort and skin integrity? Select all that apply. 1. Apply cool, moist washcloths to the affected areas 2. Keep the fingernails trimmed short to minimize skin scratching 3. Take a hot bath or shower to alleviate itching sensations 4. Use skin protectant or moisturizing cream over unbroken skin 5. Wear cotton gloves or long-sleeved clothing to avoid scratching

1. Apply cool, moist washcloths to the affected areas 2. Keep the fingernails trimmed short to minimize skin scratching 4. Use skin protectant or moisturizing cream over unbroken skin 5. Wear cotton gloves or long-sleeved clothing to avoid scratching A client with cirrhosis may experience pruritus (itching) due to buildup of bile salts beneath the skin. Clients with cirrhosis are also at an increased risk for skin breakdown due to the development of edema, which increases skin fragility and impedes wound healing, and the loss of muscle and fat tissue from pressure points (eg, heels, sacrum). The nurse encourages the client to cut the nails short, wear cotton gloves, and wear long-sleeved shirts to avoid injury to the skin from scratching (Options 2 and 5). Other comfort measures include baking soda baths; calamine lotion; and cool, wet cloths, which cool and soothe irritated skin (Options 1 and 4). Cholestyramine (Questran) may be prescribed to increase the excretion of bile salts in feces, thereby decreasing pruritus. It is packaged in powdered form, must be mixed with food (applesauce) or juice (apple juice), and should be given 1 hour after all other medications. Educational objective:A client with cirrhosis may experience pruritus (itching) due to the buildup of bile salts beneath the skin. Comfort measures include encouraging the client to cut nails short and wear long-sleeved cotton shirts and cotton gloves. Baking soda baths, calamine lotion, and cool, wet cloths also help. Cholestyramine increases the excretion of bile salts through feces, thereby decreasing itching.

The school nurse is teaching a class of 10-year-old children about prevention of dental caries. Which recommendations would be part of the nurse's teaching plan? Select all that apply. 1. Chew sugar-free gum 2. Drink fruit drinks/juices instead of sugary, carbonated beverages 3. Include milk, yogurt, and cheese in dietary intake 4. Minimize consumption of sweet, sticky foods 5. Rinse mouth with water after meals when brushing is not possible

1. Chew sugar-free gum 3. Include milk, yogurt, and cheese in dietary intake 4. Minimize consumption of sweet, sticky foods 5. Rinse mouth with water after meals when brushing is not possible Dental caries (ie, cavities) form when bacteria (eg, Streptococcus mutans) digest carbohydrates in the mouth, producing acids that break down tooth enamel and cause mineral loss. Oral hygiene and dietary intake are significant factors contributing to the development of caries. Clients should increase intake of cariostatic foods, which have an inhibitory effect on the progression of dental caries (eg, dairy products, whole grains, fruits and vegetables, sugar-free gum containing xylitol) (Options 1 and 3). Cariogenic foods increase the risk for cavities and should be avoided. These include refined, simple sugars; sweet, sticky foods such as dried fruit (eg, raisins) and candy; and sugary beverages (eg, colas and other carbonated beverages, fruit drinks/juices) (Option 4). Additional practices to prevent dental caries include: Brushing after meals Flossing at least twice a day Rinsing the mouth with water after meals or snacks (Option 5) Drinking tap water rather than bottled water (most tap water sources add fluoride to promote dental health, whereas most bottled water does not contain fluoride) Finishing meals with a high-protein food

A newborn is being evaluated for possible esophageal atresia with tracheoesophageal fistula. Which finding is the nurse most likely to observe? 1. Choking and cyanosis during feeding 2. Concave (scaphoid) abdomen 3. Diminished lung sounds 4. Projectile vomiting after feeding

1. Choking and cyanosis during feeding Educational objective: Clinical manifestations of EA/TEF include frothy saliva, coughing, choking, drooling, and a distended abdomen. Clients may also develop apnea and cyanosis while feeding. These findings must be reported to the health care provider for further evaluation.

A client with diabetes receiving peritoneal dialysis experiences chills and abdominal discomfort. The nurse assesses the client's abdomen by pressing one hand firmly into the abdominal wall. The client experiences pain when the nurse quickly withdraws the hand. The client's most recent blood glucose level is 210 mg/dL (11.65 mmol/L). What is the priority action by the nurse? 1. Collect peritoneal fluid for culture and sensitivity 2. Heat the remaining dialysate fluid and increase the dwell time 3. Place the client in high Fowler's position 4. Prepare to administer regular insulin intravenously

1. Collect peritoneal fluid for culture and sensitivity The client's chills and rebound tenderness are signs of infection that require further Peritonitis is a common but serious complication of peritoneal dialysis. Manifestations include cloudy effluent, fever, abdominal pain, and rebound tenderness. Treatment is based on culture of the peritoneal fluid.

The health care provider (HCP) orders a small bowel follow-through (SBFT) for a client. Which instructions should the nurse include when teaching the client about this test? 1. "After the test, you may notice your stools are tarry black for a few days." 2. "During the test, a series of x-rays will be taken to assess the function of the small bowel." 3. "The HCP will use an endoscope to visualize your small bowel." 4. "Your examination is scheduled for 8:00 AM. Please drink all of the polyethylene glycol by midnight."

2. "During the test, a series of x-rays will be taken to assess the function of the small bowel." An SBFT uses sequential x-ray images to visualize the structure and function of the small intestine. The client should fast for 8 hours prior to the examination. Stools may be chalky for up to 72 hours. Black, tarry stools indicate a potential gastrointestinal bleed and should be reported immediately.

The registered nurse (RN) is supervising a graduate nurse (GN) providing postoperative teaching for a male client after an inguinal hernia repair. Which statement by the GN would cause the RN to intervene? 1. "Elevate your scrotum and apply an ice bag to reduce swelling." 2. "Practice coughing to clear secretions and prevent pneumonia." 3. "Stand up to use the urinal if you have difficulty voiding." 4. "Turn in bed and perform deep breathing every 2 hours."

2. "Practice coughing to clear secretions and prevent pneumonia." After inguinal hernia repair surgery, clients should avoid coughing and heavy lifting, ambulate early, turn and deep breathe every 2 hours, and stand when voiding. Scrotal elevation and ice packs help decrease pain and swelling.

A client with end-stage liver disease is admitted for a transplant workup. The client's spouse states that the client has not stopped drinking alcohol and may be unable to quit for 6 months before the transplant. Which is the most appropriate action for the nurse to implement? 1. Ask the transplant team to place a palliative care referral so the client can learn about the option of hospice instead of transplant 2. Assess the client's motivation to make the necessary self-care changes before and after the transplant 3. Schedule a meeting to enlist the help of family members in encouraging the client to stay sober until the transplant. 4. Tell the nurse manager that the client may not be an appropriate transplant candidate

2. Assess the client's motivation to make the necessary self-care changes before and after the transplant The client with end-stage organ failure must be motivated and able to engage in complex self-care regimens before and after solid organ transplant. Concerns about the client's motivation to engage in necessary self-care requirements (eg, alcohol abstinence) require further assessment.

A client receiving total parenteral nutrition complains of nausea, abdominal pain, and excessive thirst. What is the best action for the nurse to take? 1. Assess the client's vital signs 2. Check the client's blood glucose 3. Report the findings to the health care provider 4. Slow down the rate of infusion

2. Check the client's blood glucose Hyperglycemia is a complication of TPN. Based on the client's reported symptoms related to hyperglycemia, the nurse needs to assess the client's blood sugar before implementing an intervention.

The nurse assessing a client's pain would expect the client to make which statement when describing the abdominal pain associated with appendicitis? 1. "My pain is a burning sensation in my upper abdomen." 2. "My pain is an 8 out of 10 and on my left side below my belly button." 3. "My pain is excruciating in my lower abdomen above my right hip." 4. "My pain is intermittent in my abdomen and right shoulder."

3. "My pain is excruciating in my lower abdomen above my right hip." Pain associated with acute appendicitis typically begins in the periumbilical region and migrates to the area overlying the appendix (McBurney's point). The client will attempt to decrease pain by lying still with the right leg flexed and preventing increased intraabdominal pressure (eg, avoiding coughing, sneezing, deep inhalation).

The nurse is assessing 4 clients in the emergency department. Which client should the nurse prioritize for care? 1. Client with liver cirrhosis and ascites who has increasing abdominal distension and needs therapeutic paracentesis 2. Client with new-onset ascites from a suspected ovarian mass who needs paracentesis for diagnostic studies 3. Client with ulcerative colitis who has fever, bloody diarrhea, and abdominal distension and needs an abdominal x-ray 4. Nursing home client with dementia who has stool impaction and abdominal distension and needs stool disimpaction

3. Client with ulcerative colitis who has fever, bloody diarrhea, and abdominal distension and needs an abdominal x-ray The client with ulcerative colitis who has abdominal distension, bloody diarrhea, and fever likely has toxic megacolon. This is a common, life-threatening complication of inflammatory bowel disease and is seen more frequently in ulcerative colitis than in Crohn disease. Toxic megacolon can also be associated with Clostridium difficile infection and other forms of infectious colitis. Severe colonic inflammation causes release of inflammatory mediators and bacterial products which contribute to colonic smooth muscle paralysis. Rapid colonic distension ensues, thinning the intestinal wall and making it prone to perforation. Imaging confirms the diagnosis. Educational objective:Toxic megacolon is a common, life-threatening complication of inflammatory bowel disease. Clients present with abdominal pain/distension, bloody diarrhea, fever, and signs of shock (eg, hypotension, tachycardia).

Which group of food selections would be the best choice for a client advancing to a full liquid diet 3 days after bariatric surgery? 1. Apple juice, mashed potatoes, chocolate pudding 2. Chicken broth, low-fat cheese omelet, strawberry ice cream 3. Creamy wheat cereal, blended cream of chicken soup, protein shake 4. Low-fat vanilla yogurt, smooth peanut butter, vegetable juice

3. Creamy wheat cereal, blended cream of chicken soup, protein shake Clients recovering from bariatric surgery are given small, frequent meals to prevent nausea, vomiting, and regurgitation related to overstretching of the stomach. The bariatric postoperative diet is restricted to foods that are high in nutrients (eg, protein, fiber) and low in simple carbohydrates to prevent dumping syndrome.

The nurse is developing teaching materials for a client diagnosed with ulcerative colitis. The client will receive sulfasalazine. Which of the following instructions are included in the discharge teaching plan? Select all that apply. 1. Avoid small, frequent meals 2. Can have a cup of coffee with each meal 3. Eat a low-residue, high-protein, high-calorie diet 4. Increase fluid intake to at least 2000 mL/day 5. Medication should be continued even after the resolution of symptoms 6. Take daily vitamin and mineral supplements

3. Eat a low-residue, high-protein, high-calorie diet 4. Increase fluid intake to at least 2000 mL/day 5. Medication should be continued even after the resolution of symptoms 6. Take daily vitamin and mineral supplements A low-residue, high-protein, high-calorie diet with supplemental vitamins and minerals is recommended for a client diagnosed with ulcerative colitis. The well-balanced diet includes small, frequent meals and at least 2000-3000 mL/day of fluid to maintain fluid and electrolyte balance and hydration.

The nurse is caring for a debilitated client with a percutaneous endoscopic gastrostomy (PEG) tube that was inserted 3 days ago for the long-term administration of enteral feedings and medications. While the nurse is preparing to administer the feeding, the tube becomes dislodged. What is the most appropriate intervention? 1. Insert a Foley catheter into the existing tract and inflate the balloon 2. Insert a small-bore nasointestinal tube to administer feedings and medications 3. Notify the health care provider who inserted the PEG tube 4. Reinsert the PEG tube into the existing tract immediately

3. Notify the health care provider who inserted the PEG tube Hepatic encephalopathy (HE) is a frequent complication of liver cirrhosis. Precipitating factors include hypokalemia, constipation, gastrointestinal hemorrhage, and infection. It results from accumulation of ammonia and other toxic substances in blood. Clinical manifestations of HE ranges from sleep disturbances (early) to lethargy and coma. Mental status is altered, and clients are not oriented to time, place, or person. A characteristic clinical finding of HE is presence of asterixis (flapping tremors of the hands). It is assessed by having the client extend the arms and dorsiflex the wrists. Another sign is fetor hepaticus (musty, sweet odor of the breath) from accumulated digestive byproducts. HE manifests with sleep disturbances, altered mental status, and lethargy. Asterixis and elevated ammonia are characteristic of HE.

A graduate nurse (GN) is caring for a client with acute appendicitis who is awaiting surgery. Which action by the GN would require the precepting nurse to intervene? 1. Administers morphine IV PRN for pain 2. Initiates continuous normal saline IV 3. Provides a heating pad for abdominal discomfort 4. Teaches client about prescribed strict NPO status

3. Provides a heating pad for abdominal discomfort Appendicitis is an inflammation of the appendix that often requires surgical treatment. Nurses caring for clients with appendicitis should avoid applying heat to the abdomen as this increases appendiceal swelling and the risk of appendix perforation, which is a medical emergency.

The nurse who is caring for a client with acute diverticulitis will immediately report which finding to the health care provider? 1. Abdominal pain has progressed to the left upper quadrant 2. Hemoglobin of 11.2 g/dL (112 g/L) 3. Lying on side with knees drawn up to abdomen and trunk flexed 4. White blood cell count of 12,000/mm3 (12.0 x 109/L)

1. Abdominal pain has progressed to the left upper quadrant Diverticulitis is caused by inflammation of diverticula (outpouchings) of the large intestine that can lead to abscess, perforation, peritonitis, and/or bleeding. Peritonitis is a potentially fatal complication that should be reported to the health care provider immediately.

The nurse assessing a client with an upper gastrointestinal bleed would expect the client's stool to have which appearance? 1. Black tarry 2. Bright red bloody 3. Light gray "clay-colored" 4. Small, dry, rocky-hard masses

1. Black tarry The nurse would expect a client experiencing an upper gastrointestinal (GI) bleed to have black tarry stools (melena). As blood passes through the GI tract, digestion of the blood ensues, producing the black tarry appearance. Clients with upper gastrointestinal (GI) bleed tend to have black tarry stools (melena). Lower GI bleeding will have bright red bloody stool. Blood present on surface of stool indicates hemorrhoids.

A client who has been on long-term omeprazole therapy for gastroesophageal reflux disease is admitted to the hospital for a urinary tract infection. The nurse recognizes that this client is at highest risk for which complication due to omeprazole use? 1. Clostridium difficile infection 2. Gait disturbance 3. Jaw necrosis 4. Tremor

1. Clostridium difficile infection Long-term use of PPIs (Prazoles - omeprazole, lansoprazole, pantoprazole, rebeprazole) has been associated with decreased bone density (calcium malabsorption) and increased risk for C difficile-associated diarrhea and pneumonia.

The nurse is providing nutritional teaching for a client with a new ileostomy. Which foods should the nurse instruct the client to avoid? Select all that apply. 1. Bananas 2. Broccoli with cheese 3. Multigrain bagel 4. Popcorn 5. Spaghetti with sauce

2. Broccoli with cheese 3. Multigrain bagel 4. Popcorn Foods to be avoided include: High fiber: popcorn, coconut, brown rice, multigrain bread Stringy vegetables: celery, broccoli, asparagus Seeds or pits: strawberries, raspberries, olives Edible peels: apple slices, cucumber, dried fruit

A nurse is preparing a presentation about behavioral modifications to support weight loss for clients at an obesity clinic. Which of the following points should the nurse include in the teaching plan? Select all that apply. 1. Avoid social gatherings that occur in restaurants or around meals 2. Create multiple small goals with rewards for achievement 3. Identify a list of desired outcomes not directly related to weight loss 4. Perform anxiety-reducing activities rather than using food to cope with stress 5. Utilize visual cues such as motivational quotes to encourage positive behavior

2. Create multiple small goals with rewards for achievement 3. Identify a list of desired outcomes not directly related to weight loss 4. Perform anxiety-reducing activities rather than using food to cope with stress 5. Utilize visual cues such as motivational quotes to encourage positive behavior Obesity is a health alteration with multiple causative factors (eg, genetics, diet, pathology, lifestyle choices). Successful behavioral modification is achieved by goal setting, incentivized behavior, diversion, and positive reinforcement.

The registered nurse is developing a nursing care plan for a client who has just undergone surgery for treatment of ulcerative colitis with the creation of a permanent ileostomy. What is the priority outcome for this client? 1. The client will contact the United Ostomy Association of America 2. The client will look at and touch the stoma 3. The client will read the materials provided on ostomy care 4. The client will verbalize methods to control gas and odor

2. The client will look at and touch the stoma A client with a change in body image and functioning, such as the creation of an ostomy, will need to adapt to and cope with the significant changes. Support and teaching will assist the client in overcoming psychosocial barriers to self-care; performance of a desired action is the strongest indicator of learning and acceptance.

The clinic nurse provides teaching for a client scheduled for a barium enema the next day. Which statement by the client shows a need for further instruction? 1. "I can expect chalky white stool after the procedure." 2. "I cannot eat or drink 8 hours before the procedure." 3. "I may have abdominal cramping during the procedure." 4. "I will avoid laxatives after the procedure."

4. "I will avoid laxatives after the procedure." A barium enema uses fluoroscopy with contrast to evaluate for colon abnormalities. Before the procedure, clients undergo bowel preparation using cathartics and a clear liquid diet. Laxatives after the procedure assist in expelling the contrast.

After performing a physical assessment and obtaining vital signs for a client immediately after a laparoscopic cholecystectomy, which nursing intervention is the priority? 1. Apply anti-embolism stockings 2. Assist with early ambulation 3. Offer stool softeners 4. Provide low-fat foods

1. Apply anti-embolism stockings The priority of care after a laparoscopic cholecystectomy is the prevention of complications. Early ambulation helps prevent complications by: improving breathing and reducing discomfort from the carbon dioxide used to expand the abdomen during surgery; decreasing the risk of thromboembolism; and stimulating peristalsis/bowel motility.

During assessment of a client who had major abdominal surgery a week ago, the nurse notes that the incision has dehisced and evisceration has occurred. The nurse stays with the client while another staff member gets sterile gauze and saline. How should the nurse position the client while waiting to cover the wound? 1. Low Fowler's position with knees bent 2. Prone to prevent further evisceration 3. Side-lying lateral position 4. Supine with head of the bed flat

1. Low Fowler's position with knees bent Wound evisceration is a medical emergency. The client should be placed in low Fowler's position with the knees bent to reduce tension on the open wound. The nurse should remain with the client while another staff member obtains sterile saline and gauze to cover the wound.

The nurse plans care for a pediatric client who has just undergone a cleft palate repair. Which of the following interventions should the nurse include in the plan of care? Select all that apply. 1. Assist and encourage caregivers to hold and comfort the child 2. Offer a pacifier in between feedings to promote the child's comfort 3. Position the child supine with an elevated head of bed after feedings 4. Remove elbow restraints per policy for skin and circulatory assessment 5. Use tongue blade and penlight to assess surgical site every 4 hours

1. Assist and encourage caregivers to hold and comfort the child 3. Position the child supine with an elevated head of bed after feedings 4. Remove elbow restraints per policy for skin and circulatory assessment Following cleft palate repair, the nurse should position the client supine with elevated head of bed, implement pain reduction measures, and remove elbow restraints per facility policy for skin and neurovascular assessment. To prevent damage to the surgical site, hard objects should not be placed into the mouth.

The nurse is teaching the home health client how to perform colostomy irrigation. Which client action reveals that further teaching is required? 1. Attaches an enema set to the irrigation bag, lubricates it, gently inserts it into the stoma, and holds it in place. 2. Fills irrigation container with 500-1000 mL of lukewarm tap water and flushes the irrigation tubing. 3. Hangs the irrigation container on a hook at the level of the shoulder approximately 22 inches above the stoma 4. Slowly opens the roller clamp, allowing the irrigation solution to flow, but clamps the tubing when cramping occurs

1. Attaches an enema set to the irrigation bag, lubricates it, gently inserts it into the stoma, and holds it in place. Colostomy irrigation allows the client to create a bowel regimen and to apply a dressing or smaller pouch device over the stoma. To properly irrigate the stoma, use 500-1000 mL of lukewarm water, hang the bag 18-24 inches above the stoma, use the cone-tipped irrigator to slowly infuse the solution, and allow stool to drain through the sleeve into the toilet.

The nurse understands that which of these body substances are modes of transmission for hepatitis B? Select all that apply. 1. Blood 2. Feces 3. Semen 4. Urine 5. Vaginal secretions

1. Blood 3. Semen 5. Vaginal secretions The transmission of hepatitis B occurs through parenteral or sexual contact with body fluids such as blood, semen, or vaginal secretions (mnemonic: B for body fluids).

A nurse is precepting a new graduate nurse who is caring for a client with a paralytic ileus and a Salem sump tube attached to continuous suction. The preceptor should intervene when the graduate nurse performs which interventions? Select all that apply. 1. Checks for residual every 4 hours 2. Places client in semi-Fowler's position 3. Plugs the air vent if gastric content refluxes 4. Provides mouth care every 4 hours 5. Turns off suction when auscultating bowel sounds

1. Checks for residual every 4 hours 3. Plugs the air vent if gastric content refluxes General interventions to maintain gastric suction when using a Salem sump tube include: Maintaining client in semi-Fowler's position Accurate assessment of bowel sounds Keeping the air vent (blue pigtail) open and above the level of the client's stomach Providing mouth care every 4 hours to maintain moisture of oral mucosa and promote comfort Inspecting the drainage system for patency

A nurse is caring for a client who developed paralytic ileus after a stroke. The client reports nausea, abdominal discomfort, and distension; bowel sounds are absent. Which prescription does the nurse question? 1. Hydrocodone 5/325 mg 1 tab every 4 hours PRN for moderate pain 2. Increase continuous IV normal saline rate from 75 to 100 mL/hr 3. Insert nasogastric tube and attach to wall suction 4. Ondansetron 4 mg IVP every 4 hours PRN for nausea

1. Hydrocodone 5/325 mg 1 tab every 4 hours PRN for moderate pain Opioid medications can worsen constipation and paralytic ileus and therefore should be avoided in high-risk clients (eg, stroke, post abdominal surgery).

The nurse assesses a child with intussusception. Which assessment findings require priority intervention? 1. Abdominal rigidity with guarding 2. Absence of tears in crying child with IV start 3. Blood-streaked mucous stool in diaper 4. Sausage-shaped right-sided mass on palpation

1. Abdominal rigidity with guarding Intestinal perforation and peritonitis are common complications of intestinal obstruction (eg, intussusception). Peritonitis is characterized by fever, abdominal rigidity, guarding, and rebound tenderness and is a surgical emergency.

The nurse assesses for cancer risk factors during a screening event at a gastroenterology clinic. Which of the following client statements include risk factors for esophageal cancer? Select all that apply. 1. "A few years ago, I switched from smoking cigarettes to smoking cigars 1 or 2 times a week." 2. "I am proud that I was able to lose 10 lb, but I'm still considered obese for my height." 3. "I drink 3 or 4 beers nightly to relax, but I did switch to light beer recently." 4. "I have struggled with daily episodes of acid reflux for years, especially at nighttime." 5. "I snack on a lot of salted foods like popcorn and peanuts."

1. "A few years ago, I switched from smoking cigarettes to smoking cigars 1 or 2 times a week." 2. "I am proud that I was able to lose 10 lb, but I'm still considered obese for my height." 3. "I drink 3 or 4 beers nightly to relax, but I did switch to light beer recently." 4. "I have struggled with daily episodes of acid reflux for years, especially at nighttime." Esophageal cancer is a rapidly growing malignancy of the esophageal lining. Risk factors for esophageal cancer include smoking, excessive alcohol consumption, obesity, and gastroesophageal reflux disease.

The post-anesthesia care unit nurse receives report on a client after abdominal surgery. What sounds would the nurse expect to hear when auscultating the bowel? 1. Absent bowel sounds 2. Borborygmi sounds 3. High-pitched and gurgling sounds 4. Swishing or buzzing sounds

1. Absent bowel sounds Bowel sounds following abdominal manipulation may be absent for 24-48 hours. Any disease process that causes an increase in peristalsis may cause borborygmi (loud, gurgling sounds). Swishing and humming sounds heard best with the bell of the stethoscope may be indicative of turbulent blood flow.

The nurse is preparing a client who had a Roux-en-Y gastric bypass (RYGB) for discharge from the hospital. What information should the nurse plan to include related to the prevention of dumping syndrome? 1. Meals should be small and low in carbohydrate content. 2. Fluids should be encouraged with each meal. 3. Take a multivitamin with iron and calcium supplements daily. 4. You will need to take your cobalamin injection monthly.

1. Meals should be small and low in carbohydrate content. An RYGB (anastomosis of a small gastric pouch to the Roux limb of the small intestine) has several potential complications, including dumping syndrome, iron deficiency anemia, and cobalamin deficiency. To prevent dumping syndrome, the client should consume small meals, eat a low-carbohydrate diet, and consume food and fluids 30 minutes apart.

The nurse is caring for a 2-year-old who is receiving a saline enema for treatment of intussusception. Reporting which client finding to the health care provider (HCP) is most important? 1. Passed a normal brown stool. 2. Passed a stool mixed with blood. 3. Stopped crying. 4. Vomited a third time

1. Passed a normal brown stool. Reduction of intussusception is often performed with a saline or air enema. The HCP should be notified if there is passage of a normal stool as this indicates reduction of the intussusception. All plans for surgery should be stopped and the plan of care should be modified.

The nurse is assisting with a colorectal cancer screening using the guaiac fecal occult blood test. Place the steps for completing this test in the correct sequence. All options must be used. 1. Document the results in the electronic medical record. 2. Obtain supplies, wash hands, and apply nonsterile gloves. 3. Open the back of the slide and apply 2 drops of developing solution to each box. 4. Open the slide's flap and apply 2 separate stool sample to the boxes on the slide. 5. Wait 30-60 seconds.

2. Obtain supplies, wash hands, and apply nonsterile gloves. 4. Open the slide's flap and apply 2 separate stool samples to the boxes on the slide. 3. Open the back of the slide and apply 2 drops of developing solution to each box. 5. Wait 30-60 seconds. 1. Document the results in the electronic medical record. The guaiac fecal occult blood test detects microscopic blood in the stool and is used to screen for colorectal cancer. A blue color on the Hemoccult slide paper within 30-60 seconds indicates a positive result.

A 70-year-old client is admitted to the hospital with a lower gastrointestinal bleed. After assisting the client back to bed, the nurse finds approximately 600 mL of frank red blood in the commode. The client is now pale and diaphoretic and reports dizziness. Which action should the nurse perform first? 1. Check the vital signs 2. Draw blood for hemoglobin and hematocrit 3. Lower the head of the bed 4. Maintain an IV line with normal saline

3. Lower the head of the bed Acute blood loss is a medical emergency, and the nurse needs to carry out interventions rapidly. Lowering the head of the bed or placing the client in the supine position maintains blood perfusion to the brain and other vital organs. This can be done quickly to help stabilize the client before performing other interventions. A client with significant blood loss has a medical emergency, and interventions that will hemodynamically stabilize the client should take priority.

A 78-year-old client recovering from a hip fracture tells the home health nurse, "I haven't had much of an appetite lately and have been really tired. I'm worried I'm not eating enough." Which question is the priority for the nurse to ask? 1. "Are you able to prepare your own meals?" 2. "Are you feeling lonely or depressed?" 3. "Have you lost any weight unintentionally?" 4. "How many meals do you eat each day?"

3. "Have you lost any weight unintentionally?" Educational objective:Malnutrition occurs when nutrient intake is insufficient for body requirements. Nurses evaluating clients for malnutrition should first assess for unintentional weight loss, an important indicator of malnourishment. Afterwards, contributing factors of malnutrition (eg, functional status, mood alteration, diet) should be evaluated.

The nurse provides discharge instructions to a client one day after laparoscopic cholecystectomy. Which statement by the client indicates that further teaching is required? 1. "I can resume a regular diet but will avoid fatty foods for several weeks after surgery." 2. "I can return to work within a week of surgery." 3. "I will report to the health care provider if my temperature is higher than 101 F (38.3 C)." 4. "Tomorrow I can remove the puncture site bandages and take a bath."

4. "Tomorrow I can remove the puncture site bandages and take a bath." Educational objective:A laparoscopic cholecystectomy enables clients to recover and resume normal activities more quickly than an open surgical technique. Clients should be taught to remove surgical bandages the day after surgery; showering may be resumed at this time. Clients should increase activity slowly and eat a low-fat diet.

A client is admitted with severe acute pancreatitis. While obtaining the client's blood pressure, the nurse notices a carpal spasm. What laboratory result would the nurse assess in response to this symptom? 1. Decreased albumin 2. Elevated troponin 3. Hyperkalemia 4. Hypocalcemia

4. Hypocalcemia Complications of acute severe pancreatitis include hyperglycemia, hypocalcemia, hypovolemia, and ARDS. Trousseau's (carpal spasm) and Chvostek's (facial twitching) signs are an indication of hypocalcemia from the decrease in threshold for contraction.

A nurse in the neonatal intensive care unit discovers a cyanotic newborn with excessive frothy mucus in the mouth. What should be the nurse's first action? 1. Administer 100% oxygen 2. Auscultate the lungs 3. Place infant in knee-chest position 4. Suction the infant's mouth

4. Suction the infant's mouth The initial nursing action for a client experiencing cyanosis and excess oral secretions is oropharyngeal suctioning to ensure airway patency

A client comes to the clinic for a follow-up visit after a Billroth II surgery (gastrojejunostomy). The client reports occasional episodes of sweating, palpitations, and dizziness 30 minutes after eating. Which nursing action is most appropriate? 1. Check serum blood glucose for hypoglycemia. 2. Ensure that the client consumes fluids with meals. 3. Take the client's blood pressure while lying and standing. 4. Teach the client to lie down after eating.

4. Teach the client to lie down after eating.

The nurse reinforces teaching to a female client about taking misoprostol to prevent stomach ulcers. Which statement by the client would prompt further instruction? 1. "I can take this medication with food if it hurts my stomach." 2. "I must use a reliable form of birth control while taking this medication." 3. "I should continue to take my ibuprofen as prescribed." 4. "I will take this medicine with an antacid to decrease stomach upset."

4. "I will take this medicine with an antacid to decrease stomach upset." Antacids, especially those that contain magnesium (eg, Gaviscon), can increase the adverse effects of misoprostol (eg, diarrhea, dehydration). If clients require therapy with antacids, they should choose one that does not contain magnesium (eg, calcium carbonate [Tums]) and contact the health care provider if adverse effects occur (Option 4). Misoprostol prevents gastric ulcers in clients receiving long-term nonsteroidal anti-inflammatory drug therapy. It should not be taken with antacids but can be taken with food to reduce gastrointestinal upset. Women of childbearing age should be educated on using reliable birth control methods as misoprostol can induce labor.

A 3-month-old infant has irritability, facial edema, a 1-day history of diarrhea with adequate oral intake, and seizure activity. During assessment, the parents state that they have recently been diluting formula to save money. Which is the most likely cause for the infant's symptoms? 1. Hypernatremia due to diarrhea 2. Hypoglycemia due to dilute formula intake 3. Hypokalemia due to excess gastrointestinal output 4. Hyponatremia due to water intoxication

4. Hyponatremia due to water intoxication Water intoxication (water overload) resulting in hyponatremia may occur in infants when formula is diluted to "stretch" the feeding to save money. Hyponatremia may also result from ingestion of plain water (eg, caregiver attempting to rehydrate an infant who has been ill). Infants have immature renal systems with a low glomerular filtration rate, which decreases their ability to excrete excess water and makes them susceptible to water intoxication. Symptoms of hyponatremia include irritability, lethargy, and, in severe cases, hypothermia and seizure activity. Breast milk and/or formula are the only sources of hydration an infant needs for the first 6 months of life. Formula should be prepared per the manufacturer's instructions. Infants are susceptible to hyponatremia secondary to water intoxication, which can present with neurological symptoms (eg, lethargy, irritability, seizures). Breast milk and/or formula provide sufficient hydration for the first 6 months of life. Formula should not be diluted to save money.

The nurse is assessing a client who had an esophagogastroduodenoscopy 3 hours ago. The client is reporting increasing abdominal pain. Which clinical finding requires an immediate report to the health care provider? 30 minutes ago Client returned from GI lab, drowsy but oriented x4. Client rates throat pain as 2 on a scale of 0-10 and new-onset abdominal pain as 4 on a scale of 0-10. Family at bedside. , RN Vital signs2 hours ago1 hour ago30 minutes agoNowTemperature98.4 F (36.9 C)98.8 F (37.1 C)99.4 F (37.4 C)100.6 F (38.1 C)Blood pressure116/78 mm Hg110/75 mm Hg112/74 mm Hg108/72 mm HgHeart rate85/min87/min95/min112/minRespirations18/min17/min18/min22/minSpO298%98%98%98% 1. Blood pressure 108/72 mm Hg 2. Gag reflex has not returned 3. Sore throat when swallowing 4. Temperature 100.6 F (38.1 C)

4. Temperature 100.6 F (38.1 C) Perforation of the gastrointestinal tract is a life-threatening complication of esophagogastroduodenoscopy that can lead to peritonitis and sepsis. Signs of perforation (eg, fever, increasing pain/tenderness, tachycardia, tachypnea) require immediate notification of the health care provider.

The nurse is caring for a client with acute pancreatitis admitted 2 days ago. Which assessment finding is most concerning? 1. Blood glucose levels for the past 24 hours are ≥250 mg/dL (13.9 mmol/L)(14%) 2. Client is lying with knees drawn up to the abdomen to alleviate pain(9%) 3. Five large, liquid stools that are yellow and foul-smelling(13%) 4. Temperature of 102.2 F (39 C) with increasing abdominal pain

4. Temperature of 102.2 F (39 C) with increasing abdominal pain Educational objective:Acute pancreatitis may cause severe midepigastric abdominal pain, elevated blood glucose levels, and steatorrhea. The nurse should watch closely for high fever, increasing abdominal pain, and leukocytosis as these findings may indicate infection of the necrosed pancreas or pancreatic abscess formation.

A client is 1-day postoperative abdominoplasty and is discharged to go home with a Jackson-Pratt (JP) closed-wound system drain in place. The nurse teaches the client how to care for the drain and empty the collection bulb. Which statement indicates that the client needs further instruction? 1. "I'll empty the JP bulb when it is totally full so that I don't have to unplug it so many times." 2. "I'll pull the plug on the JP bulb and pour the drainage into the measurable specimen cup." 3. "I'll squeeze the JP bulb from side-to-side as I hold it in my hand." 4. "While the JP bulb is totally compressed, I'll clean the spout with alcohol and replace the plug."

1. "I'll empty the JP bulb when it is totally full so that I don't have to unplug it so many times." A closed-wound drain device (eg, JP, Hemovac) is used to prevent fluid buildup at the surgical wound site and promote healing. Empty the device every 4-12 hours unless it is 1/2 to 2/3 full before then. Drainage tube patency and negative pressure in the reservoir (bulb) must be maintained to provide adequate drainage.

A healthy 50-year-old client asks the nurse, "What must I do in preparation for my screening colonoscopy?" Which statements by the nurse correctly answer the client's question? Select all that apply. 1. "No food or drink is allowed 8 hours prior to the test." 2. "Prophylactic antibiotics are taken as prescribed." 3. "Smoking must be avoided after midnight." 4. "The day prior to the procedure your diet will be clear liquids." 5. "You will drink polyethylene glycol as directed the day before."

1. "No food or drink is allowed 8 hours prior to the test." 4. "The day prior to the procedure your diet will be clear liquids." 5. "You will drink polyethylene glycol as directed the day before." Instructions for clients scheduled for a colonoscopy include a clear liquid diet the day before the procedure, avoiding any food or liquids (nothing by mouth) 8-12 hours prior to the examination, and taking the bowel-cleansing agent as prescribed.

The clinic nurse educator is developing a teaching plan for the following 6 clients. The nurse should instruct which client to avoid the Valsalva maneuver when defecating? Select all that apply. 1. 22-year-old man with a head injury sustained during a college football game 2. 30-year-old woman recently hospitalized for reconstructive augmentation mammoplasty 3. 56-year-old man 2 weeks post myocardial infarction 4. 68-year-old woman recently diagnosed with pancreatic cancer 5. 74-year-old man with portal hypertension related to alcohol-induced cirrhosis 6. 82-year-old woman 1 week post cataract surgery

1. 22-year-old man with a head injury sustained during a college football game 3. 56-year-old man 2 weeks post myocardial infarction 5. 74-year-old man with portal hypertension related to alcohol-induced cirrhosis 6. 82-year-old woman 1 week post cataract surgery. Educational objective:The Valsalva maneuver is contraindicated in the client diagnosed with increased intracranial pressure, stroke, head injury, heart disease, glaucoma, eye surgery, abdominal surgery, and liver cirrhosis.

During the admission assessment of a client with a small-bowel obstruction, the nurse anticipates which clinical manifestations? Select all that apply. During the admission assessment of a client with a small-bowel obstruction, the nurse anticipates which clinical manifestations? Select all that apply. 1. Abdominal distension 2. Absolute constipation 3. Colicky abdominal pain 4. Frequent vomiting 5. Pain during defecation

1. Abdominal distension 3. Colicky abdominal pain 4. Frequent vomiting Common symptoms of small-bowel obstruction include rapid onset of nausea and vomiting, colicky intermittent abdominal pain, and abdominal distension. Absolute constipation and lack of flatus are usually seen with large-bowel obstruction. Initial treatment of an obstruction includes placing the client on NPO status, inserting a nasogastric tube, administering IV fluids, and instituting pain control measures.

An older client comes to the outpatient clinic for a routine physical examination and health screening. Which findings does the nurse recognize as possible indications of colorectal cancer? Select all that apply. 1. Abdominal pain 2. Blood in the stools 3. Change in bowel habits 4. Low hemoglobin level 5. Unexplained weight loss

1. Abdominal pain 2. Blood in the stools 3. Change in bowel habits 4. Low hemoglobin level 5. Unexplained weight loss Clients over age 50 should receive routine colorectal cancer screening for symptoms such as blood in the stool, anemia, abdominal discomfort, change in bowel habits, and weight loss. Symptoms result from intestinal polyps or tumors that cause intestinal bleeding, obstruction, and impaired intestinal absorption.

A hospitalized client with acute pancreatitis has nausea, vomiting, epigastric pain, and tachycardia. Laboratory results show elevated serum lipase levels. Which interventions would the nurse anticipate being prescribed for the client? Select all that apply. 1. Administer hydromorphone IV PRN for pain 2. Administer intravenous fluids 3. Insert a nasogastric tube for nasogastric suction 4. Maintain client in a supine position, with head of bed flat 5. Provide small, frequent, high-carbohydrate, high-calorie meals

1. Administer hydromorphone IV PRN for pain 2. Administer intravenous fluids 3. Insert a nasogastric tube for nasogastric suction. The major goals in acute pancreatitis are symptom management (eg, opioids, NPO status, nasogastric suction) and monitoring and prevention of complications (eg, IV fluids), giving the pancreas time to heal.

The nurse is caring for a client with liver cirrhosis who was admitted for cellulitis of the leg. Which assessments would the nurse perform to determine if the client's condition has progressed to hepatic encephalopathy? Select all that apply. 1. Ask if the client knows what day it is 2. Ask the client to extend the arms 3. Assess for telangiectasia (spider nevi) 4. Determine if the conjunctiva is jaundiced 5. Note amylase and lipase serum levels

1. Ask if the client knows what day it is 2. Ask the client to extend the arms A PEG tube's tract begins to mature in 1-2 weeks and is fully established in 4-6 weeks. Tube dislodgement <7 days from placement requires surgical or endoscopic replacement. Attempting to reinsert a tube through an immature tract can result in improper placement into the peritoneal cavity, leading to peritonitis and sepsis.

The nurse is teaching about the importance of dietary fiber at a community health fair. Which health benefits of consuming a fiber-rich diet should the nurse include in the teaching plan? Select all that apply. 1. Helps prevent colorectal cancer 2. Improves glycemic control 3. Promotes weight loss 4. Reduces risk of vascular disease 5. Regulates bowel movements

1. Helps prevent colorectal cancer 2. Improves glycemic control 3. Promotes weight loss 4. Reduces risk of vascular disease 5. Regulates bowel movements Dietary fiber increases stool bulk and makes stool softer and easier to pass. A fiber-rich diet helps prevent constipation; decreases risk of colorectal cancer; promotes weight loss; improves blood glucose control; and decreases serum cholesterol levels, which reduces the risk of coronary artery disease and stroke

A nurse is evaluating the teaching of weight reduction strategies to a client with obesity. Which of the following statements indicate that the client understands the teaching? Select all that apply. 1. "Fruit juice is a good substitute for soda." 2. "I will aim to lose 1-2 lb (0.45-0.91 kg) per week." 3. "I will keep healthy snacks on hand in case I get hungry." 4. "I will skip breakfast to save calories for later in the day." 5. "I will take the stairs instead of the elevator."

2. "I will aim to lose 1-2 lb (0.45-0.91 kg) per week." 3. "I will keep healthy snacks on hand in case I get hungry." 5. "I will take the stairs instead of the elevator." To promote effective weight loss, the nurse teaches the client to eliminate sugar-containing beverages; aim to lose about 1-2 lb (0.45-0.91 kg) per week; eat small, frequent meals; incorporate daily exercise with increasing intensity; and get about 7-9 hours of sleep per night.

A parent brings a 6-month-old child to the primary health care provider after the child abruptly started crying and grabbing intermittently at the abdomen. The client's stool has a red, currant jelly appearance. What intervention does the nurse anticipate? 1. Administer epoetin alfa (erythropoietin) 2. Give air (pneumatic) enema 3. Have the parent give 2 ounces of extra juice a day for constipation 4. Perform hemoccult test on stool

2. Give air (pneumatic) enema Intussusception (the intestine telescoping into itself) causes intermittent cramping and progressive abdominal pain, inconsolable crying, and currant jelly stool (from blood or mucus). It is often treated successfully with an air enema.

The nurse is caring for a client with right upper quadrant pain and jaundice. The client's alanine aminotransferase /aspartate aminotransferase (ALT/AST) levels are 7 times the normal values. What questions would be most helpful regarding the etiology for these findings? Select all that apply. 1. Do you have black tarry stool? 2. Do you use intravenous (IV) illicit drugs? 3. How much alcohol do you typically drink? 4. Were you recently immunized for pneumonia? 5. What over-the-counter drugs do you take?

2. Do you use intravenous (IV) illicit drugs? 3. How much alcohol do you typically drink? 5. What over-the-counter drugs do you take? ALT/AST are enzymes indicating liver injury. Besides the obvious viral hepatitis, it can result from excess chronic alcohol intake or some over-the-counter drugs, including acetaminophen

The nurse is providing postoperative care to a client returning from a hemorrhoidectomy. Which action is the priority for the nurse to perform? 1. Administer docusate and teach the client to avoid straining during defecation 2. Give pain medications and instructions related to pain control 3. Remove the rectal dressing and check the client for bleeding 4. Teach the client how to self-administer a sitz bath 2-3 times daily

2. Give pain medications and instructions related to pain control Post-hemorrhoidectomy pain is excruciating. Providing pain relief and preventing constipation are the primary goals for these clients. Sitz baths should begin 1-2 days postoperatively. Hemorrhoids may recur with increased anorectal pressure. Therefore, clients should maintain a high-fiber diet, use stool softeners, and drink adequate fluids (at least 1500 mL/day) to prevent constipation.

The nurse is assessing an infant with intussusception. Which of the following clinical findings should the nurse expect? Select all that apply. 1. Palpable olive-shaped mass in epigastrium 2. Palpable sausage-shaped abdominal mass 3. Projectile vomiting without visualized blood 4. Screaming and drawing of the knees up to the chest 5. Stool mixed with blood and mucus

2. Palpable sausage-shaped abdominal mass 4. Screaming and drawing of the knees up to the chest 5. Stool mixed with blood and mucus Classic symptoms of intussusception include sudden, crampy abdominal pain; a palpable sausage-shaped abdominal mass; "currant jelly" stools; inconsolable crying with the knees drawn up to the chest; and bilious, nonprojectile vomiting. An olive-shaped mass is characteristic of pyloric stenosis. Projectile vomiting is frequently associated with pyloric stenosis or increased intracranial pressure.

The nurse is caring for an infant diagnosed with Hirschsprung disease who is awaiting surgery. Which assessment finding requires the nurse's immediate action? 1. Abdominal distension with no change in girth for 8 hours 2. Did not pass meconium or stool within 48 hours after birth 3. Episode of foul-smelling diarrhea and fever 4. Excessive crying and greenish vomiting

3. Episode of foul-smelling diarrhea and fever Enterocolitis, a potentially fatal complication of Hirschsprung disease, is characterized by explosive, foul-smelling diarrhea; fever; and worsening abdominal distension.

The nurse is caring for a client who has undergone a colonoscopy. Which client assessment finding should most concern the nurse? 1. Abdominal cramping 2. Frequent, watery stools 3. Positive rebound tenderness 4. Recurring flatus

3. Positive rebound tenderness The complication risks of a colonoscopy are perforation and rectal bleeding. Abdominal cramping, flatus, and watery stool are expected findings. Perforation can lead to peritonitis, with positive rebound tenderness, guarding, abdominal distension, tenesmus, and/or boardlike (rigid) abdomen.

The nurse prepares to assess a newly admitted client diagnosed with chronic alcohol abuse whose laboratory report shows a magnesium level of 1.0 mEq/L (0.5 mmol/L). Which assessment finding does the nurse anticipate? 1. Constipation and polyuria 2. Increased thirst and dry mucous membranes 3. Leg weakness and soft, flabby muscles 4. Tremors and brisk deep-tendon reflexes

4. Tremors and brisk deep-tendon reflexes Clients who abuse alcohol often have low magnesium levels that manifest as ventricular arrhythmias and/or neuromuscular excitability (similar to hypocalcemia), which includes tremors, positive Chvostek and Trousseau signs, hyperactive reflexes, and seizures.

The nurse is evaluating a client with liver cirrhosis who received IV albumin after a paracentesis to drain ascites. Which assessment finding indicates that the albumin has been effective? 1. Abdominal circumference reduced from admission recording 2. Flapping tremor no longer visible with arm extension 3. Shortness of breath no longer experienced in supine position 4. Vital signs remain within the client's normal parameters

4. Vital signs remain within the client's normal parameters Clients undergoing paracentesis to alleviate symptoms related to ascites are at risk for hypotension due to changes in abdominal pressure. IV albumin increases intravascular fluid volume and may be used to prevent hypotension associated with paracentesis.

An overweight toddler is diagnosed with iron deficiency anemia. Which is the most likely explanation for the anemia? 1. Excessive intake of meat products 2. Excessive intake of milk 3. Gastrointestinal blood loss 4. Impaired iron transfer from the mother

Iron deficiency anemia is the most common chronic nutritional disorder in children. There are many risk factors for iron deficiency, including insufficient dietary intake, premature birth, delayed introduction of solid food, and consumption of cow's milk before age 1 year. One common cause in toddlers is excessive milk intake, over 24 oz/day. In addition to becoming overweight, toddlers who consume too much milk develop iron deficiency due to the likely exclusion of iron-rich foods in favor of milk, a poor source of available iron. Treatment of iron deficiency anemia includes oral iron supplementation and increased consumption of iron-rich foods (eg, leafy green vegetables, red meats, poultry, dried fruit, fortified cereal). It is also important to limit milk intake (16-24 oz/day) in toddlers to ensure a balanced diet. Educational objective:Iron deficiency anemia is the most common nutritional disorder in children. Risk factors include premature birth, cow's milk before age 1 year, and excessive milk intake in toddlers. Prevention and treatment are achieved through proper nutrition (eg, meat, leafy green vegetables, fortified cereal) and supplementation.

A nurse is evaluating a client's understanding about infant formula preparation. Which of the following client statements indicate proper understanding? Select all that apply. 1. "I can add extra water to powdered formula if it seems that my baby wants to feed longer." 2. "I can heat formula in the microwave for less than 1 minute." 3. "I must wash the top of concentrated formula cans before opening." 4. "Leftover milk in the bottle may be refrigerated and used at a later feeding." 5. "Unused, prepared formula should be kept in the refrigerator and discarded after 48 hours."

3. "I must wash the top of concentrated formula cans before opening." 5. "Unused, prepared formula should be kept in the refrigerator and discarded after 48 hours." Infant formula should never be overdiluted, overconcentrated, or microwaved. Unused, prepared formula should be stored in the refrigerator and, if unused, discarded after 48 hours. Formula left over in a bottle after feeding should be discarded.

The nurse provides discharge instructions to a client with cirrhosis who has portal hypertension, ascites, and esophageal varices. Which statement by the client indicates that the teaching was effective? 1. "I may have one alcoholic drink a day, but no more." 2. "I may take aspirin instead of acetaminophen for fever or pain." 3. "I should avoid straining while having a bowel movement." 4. "I should eat a protein- and sodium-restricted diet."

3. "I should avoid straining while having a bowel movement." Clients with cirrhosis should eat a high-calorie, high-carbohydrate, low-sodium, and low-fat diet; moderate protein intake is recommended. They should avoid hepatotoxic substances (eg, alcohol, acetaminophen) and medications (NSAIDs) that increase bleeding risk and reduce activities that increase intraabdominal pressure.

The nurse is reviewing anticipatory guidance with the parents of a 6-month-old infant with phenylketonuria. Which statements by the nurse are appropriate? Select all that apply. 1. "A low-phenylalanine diet is required." 2. "Meat and dairy products should not be introduced into the diet." 3. "Phenylketonuria is self-limiting and usually resolves by adulthood." 4. "Special infant formula is required." 5. "Tyrosine should be removed from the diet."

1. "A low-phenylalanine diet is required." 2. "Meat and dairy products should not be introduced into the diet." 4. "Special infant formula is required." Phenylketonuria requires lifetime dietary restrictions. Infants should be given special formulas (eg, Lofenalac). For children and adults, high-phenylalanine foods (eg, meats, eggs, milk) should be restricted and replaced with protein substitutes.

Which statement made by the client demonstrates a correct understanding of the home care of an ascending colostomy? 1. "I will avoid eating foods such as broccoli and cauliflower." 2. "I will empty the pouch when it is one-half full of stool." 3. "I will irrigate the colostomy to promote regular bowel movements." 4. "I will restrict my fluid intake to 2,000 milliliters of fluid a day.

1. "I will avoid eating foods such as broccoli and cauliflower." The stool changes from liquid to more solid as it passes through the colon. Proper care of the ostomy and pouching device in clients with a colostomy includes ensuring sufficient fluid intake, preventing gas and odor, and changing the pouching system when it becomes one-third full to prevent leaks.

The nurse receives new prescriptions for a client with right lower quadrant pain and suspected acute appendicitis. Which prescription should the nurse implement first? 1. Administer 0.25 mg hydromorphone IV push for pain 2. Draw blood for complete blood count and electrolyte levels 3. Initiate IV access and infuse normal saline 100 mL/hr 4. Obtain urine specimen for urinalysis

3. Initiate IV access and infuse normal saline 100 mL/hr Nurses caring for clients with appendicitis should prioritize client care according to the ABCs (ie, airway, breathing, circulation). Initiating IV crystalloids (eg, normal saline) is a priority action that prevents circulatory collapse resulting from fluid losses (eg, vomiting, diarrhea) and NPO status.

The nurse admits a client with cirrhosis who has an upper gastrointestinal bleed from suspected gastroesophageal varices. Which new prescription should the nurse question? 1. Administer pantoprazole IV piggyback every 12 hours 2. Initiate continuous octreotide IV infusion 3. Insert and maintain a nasogastric tube 4. Maintain NPO status except for PO medications

3. Insert and maintain a nasogastric tube Gastroesophageal varix rupture/hemorrhage is a potentially lethal complication of cirrhosis that may occur from increased portal venous pressure (eg, coughing) and mechanical injury (eg, nasogastric tube insertion). The nurse should question prescriptions for activities that increase the risk of such rupture.

A 2-year-old in the emergency department is suspected of having intussusception. Which assessment finding should the nurse expect? 1. Black, sticky stools 2. Greasy, foul-smelling stools 3. Stools mixed with blood and mucus 4. Thin, "ribbon-like" stools

3. Stools mixed with blood and mucus The classic symptom triad of intussusception is abdominal pain, "currant jelly" stools, and a sausage-shaped abdominal mass. However, it is more common for clients to have episodes of sudden abdominal pain, inconsolable crying, and vomiting followed by periods of normal behavior.

An 80-year-old client has been hospitalized with pneumonia and malnutrition. Physical assessment findings include weakness and decreased muscle mass. Which finding best indicates that the client is responding to treatment? 1. Client consuming 90% of each meal 2. Serum albumin of 3.6 g/dL (36 g/L) 3. Weight gain of 2 lb (0.9 kg) in 2 weeks 4. White blood cell count of 15,000/mm 3 (15.0 × 109/L)

3. Weight gain of 2 lb (0.9 kg) in 2 weeks The best assessment finding for indicating improved nutritional status is a steady weight gain over a specified period. Serum prealbumin is a faster and more reliable indicator of current nutritional status than serum albumin.

The nurse is educating the parents of a 6-month-old about introducing solid foods into the infant's diet. Which parental statement indicates a need for further teaching? 1. "I can introduce soft finger foods before my child has teeth." 2. "I can offer a variety of foods within the first week of introducing solids." 3. "I can prepare rice cereal with formula, breast milk, or water." 4. "I can save money by preparing baby food at home instead of buying it."

2. "I can offer a variety of foods within the first week of introducing solids." The introduction of solid foods generally occurs at age 4-6 months. When introducing new foods, parents should allow several (eg, 4-7) days between each new food to observe for any reactions to a specific food. Allergic responses often worsen with subsequent exposure, so it is a priority to identify food allergies early. Infants can begin eating solid foods at age 4-6 months. Parents should introduce each new food separately and wait several (eg, 4-7) days between each one to identify any food allergies. After starting with iron-fortified cereal (mixed with formula, breast milk, or water), parents can begin offering soft fruits and vegetables and simple finger foods.

The nurse is teaching a client with newly diagnosed lactase deficiency about dietary management. Which statements by the client indicate a correct understanding of this condition? Select all that apply. 1. "I can still eat cheese and yogurt as long as they don't make me feel sick." 2. "I should take a daily calcium and vitamin D supplement." 3. "Most dairy products should be eliminated from my diet, but ice cream is okay." 4. "My lactase enzyme supplement should be taken with meals containing dairy." 5. "This means that I have developed an allergy to milk."

1. "I can still eat cheese and yogurt as long as they don't make me feel sick." 2. "I should take a daily calcium and vitamin D supplement." 4. "My lactase enzyme supplement should be taken with meals containing dairy." Clients with lactase deficiency can prevent unpleasant gastrointestinal symptoms by avoiding lactose-containing dairy products (eg, milk, ice cream), eating cheese or yogurt in moderation, and supplementing with lactase enzymes. Vitamin D and calcium supplementation is also recommended.

The nurse prepares to admit a client with worsening cirrhosis who is on the waiting list for a liver transplant. Based on the client's electronic health record, the nurse anticipates which assessment findings? Select all that apply. Click on the exhibit button for additional information. 1. Ascites 2. Bruising 3. Constipation 4. Itching 5. Lethargy

1. Ascites 2. Bruising 4. Itching 5. Lethargy Laboratory abnormalities common in liver failure include low serum albumin (causes ascites), elevated INR (increases risk for bruising and bleeding), elevated serum ammonia (causes lethargy and confusion), and increased bilirubin (causes jaundice and itching).

The nurse is teaching a class on nutrition and feeding practices for young children. What should the nurse recommend as the best snack for a toddler? 1. ½ cup orange juice 2. Dry, sweetened cereal 3. Raw carrot sticks 4. Slices of cheese

4. Slices of cheese Food for young children should contain valuable nutrients and pose little risk of choking or foodborne infection. An example of a healthy snack for a toddler is a slice of cheese

The nurse is reinforcing education to a client with irritable bowel syndrome who is experiencing diarrhea. Which of these meals selected by the client indicates an understanding of diet management? 1. Beans, yogurt, and a fruit cup 2. Beef, broccoli, and a glass of wine 3. Eggs, a bagel, and black coffee 4. Steak, tomato basil soup, and cornbread

4. Steak, tomato basil soup, and cornbread Educational objective:Irritable bowel syndrome is a chronic condition characterized by altered intestinal motility, causing abdominal discomfort with diarrhea and/or constipation. Clients can manage symptoms by avoiding gas-producing foods (eg, broccoli), caffeine, alcohol, and gastrointestinal irritants (eg, high-fructose corn syrup, spices, dairy products) and by increasing fiber.

The nurse teaching the parents of a child diagnosed with cystic fibrosis will advise the parents to choose foods that satisfy which recommended diet? 1. Gluten-free with added protein 2. High calorie, high protein, high fat 3. High protein, low fat, low phosphate 4. High protein, low fat, low sodium

2. High calorie, high protein, high fat In cystic fibrosis (CF), a protein responsible for transporting sodium and chloride is defective and causes the secretions from the exocrine glands to be thicker and stickier than normal. These abnormal secretions plug smaller airway passages and ducts in the gastrointestinal (GI) tract. The thick secretions block pancreatic ducts, resulting in a deficient amount of pancreatic enzymes entering the bowel to aid in digestion and nutrient absorption. Clients require multiple vitamin supplements and supplemental pancreatic enzymes that are administered with meals. To meet the growth needs of clients with CF, a diet high in calories, fat, and protein is required. Educational objective:Cystic fibrosis causes damage to the GI tract and pancreas, leading to impaired absorption of nutrients and resulting growth deficits. Clients must consume a diet high in calories, fat, and protein.

The nurse is reinforcing strategies to manage symptoms for a client with irritable bowel syndrome. Which of the following instructions should the nurse include? Select all that apply. 1. Consume only clear liquids with severe symptoms 2. Keep a record of symptoms, diet, and stress levels 3. Limit dietary intake of gas-producing foods like legumes 4. Perform aerobic exercise at least three times weekly 5. Reduce the amount of caffeine consumed each day

2. Keep a record of symptoms, diet, and stress levels 3. Limit dietary intake of gas-producing foods like legumes 4. Perform aerobic exercise at least three times weekly 5. Reduce the amount of caffeine consumed each day

The hospitalized client with anorexia nervosa is started on nutrition via enteral and parenteral routes. Which client assessment is the most important for the nurse to check during the first 24-48 hours of administration? 1. Serum albumin level and body weight 2. Serum potassium and phosphate 3. Symptoms of dumping syndrome 4. White blood cell count and neutrophils

2. Serum potassium and phosphate Educational objective: Refeeding syndrome is a potentially fatal complication of nutritional rehabilitation in chronically malnourished clients. Electrolytes, especially phosphorous, potassium, and magnesium, must be monitored frequently during the first few days of nutritional replenishment.

The nurse is reinforcing teaching to a client with a hiatal hernia. Which statement by the client indicates that further teaching is needed? 1. "I need to raise the head of my bed on blocks by at least 6 inches." 2. "I will remain sitting up for several hours after I eat any food." 3. "If my reflux and abdominal pain don't improve, I might need surgery." 4. "Losing weight may reduce my reflux, so I plan to take a weight-lifting class."

4. "Losing weight may reduce my reflux, so I plan to take a weight-lifting class." Hiatal hernia is characterized by abnormal movement of the stomach and/or esophagogastric junction into the chest due to diaphragmatic weakness. Nurses educating clients with hiatal hernias about symptom management should instruct them to avoid activities that increase abdominal pressure (eg, weight lifting), sleep with the head of the bed elevated, and remain upright for several hours after meals.

The nurse is reinforcing discharge instructions with a client following a partial gastrectomy. Which of the following instructions should the nurse include to prevent dumping syndrome? Select all that apply. 1. Add high-protein foods to diet 2. Consume high-carbohydrate meals 3. Eat small, frequent meals 4. Increase intake of fluids with meals 5. Lie down after eating

1. Add high-protein foods to diet 3. Eat small, frequent meals 5. Lie down after eating Dumping syndrome is a complication of gastrectomy. To delay gastric emptying and reduce the risk of dumping syndrome, clients should consume meals low in carbohydrates and high in fiber, proteins, and fats; avoid fluids during meals; eat small, frequent meals; and lie down after eating.

Which instructions should the nurse include when providing discharge teaching to a client with peptic ulcer disease due to Helicobacter pylori infection? Select all that apply. 1. "Avoid foods that may cause epigastric distress such as spicy or acidic foods." 2. "It is best if you refrain from consuming alcohol products." 3. "Report black tarry stools to your health care provider immediately." 4. "Take your amoxicillin, clarithromycin, and omeprazole for the next 14 days." 5. "You may take over-the-counter drugs such as aspirin if you have mild epigastric pain."

1. "Avoid foods that may cause epigastric distress such as spicy or acidic foods." 2. "It is best if you refrain from consuming alcohol products." 3. "Report black tarry stools to your health care provider immediately." 4. "Take your amoxicillin, clarithromycin, and omeprazole for the next 14 days." Client teaching related to peptic ulcer disease (PUD) includes lifestyle changes (eg, dietary modifications, stress reduction), PUD complications, and medication administration. Helicobacter pylori infection and treatment with nonsteroidal anti-inflammatory drugs (NSAIDs) are risk factors for complicated PUD. H pylori treatment includes antibiotics and proton-pump inhibitors for acid suppression. The recommended initial treatment is 7-14 days of triple-drug therapy with omeprazole (Prilosec), amoxicillin, and clarithromycin (Biaxin). Clients with peptic ulcer disease should avoid NSAIDs, smoking, and excess use of alcohol or caffeine.

The nurse is reinforcing teaching about ulcer prevention with a client newly diagnosed with peptic ulcer disease. Which of the following client statements indicate appropriate understanding of teaching? Select all that apply. 1. "I need to avoid taking medicines like ibuprofen without a prescription." 2. "I should avoid drinking excess coffee or cola." 3. "I should enroll in a smoking cessation program." 4. "I should reduce or eliminate my intake of alcoholic beverages." 5. "I will eliminate whole wheat foods, like breads and cereals, from my diet."

1. "I need to avoid taking medicines like ibuprofen without a prescription." 2. "I should avoid drinking excess coffee or cola." 3. "I should enroll in a smoking cessation program." 4. "I should reduce or eliminate my intake of alcoholic beverages." Peptic ulcer disease (PUD) is a gastrointestinal illness caused by breaks in the gastrointestinal mucosa, leading to ulcer formation. To reduce ulcer formation risk, clients with PUD should be instructed to stop smoking; avoid chronic NSAID use; avoid meals or snacks before sleeping; and limit alcohol and caffeine consumption.

The nurse is providing discharge teaching to a client newly diagnosed with ulcerative colitis. Which of the following statements by the client indicate that teaching has been effective? Select all that apply. 1. "I need to eat a diet high in calories and protein so that I avoid losing weight." 2. "I need to take multivitamins containing calcium daily." 3. "I should avoid consuming alcoholic beverages." 4. "I should drink at least 2 liters of water daily and more when I have diarrhea." 5. "I will keep a symptom journal to note what I eat and drink during the day."

1. "I need to eat a diet high in calories and protein so that I avoid losing weight." 2. "I need to take multivitamins containing calcium daily." 3. "I should avoid consuming alcoholic beverages." 4. "I should drink at least 2 liters of water daily and more when I have diarrhea." 5. "I will keep a symptom journal to note what I eat and drink during the day." Ulcerative colitis (UC) is an inflammatory bowel disease that is managed with dietary interventions in addition to medication. Clients with UC should maintain a high-calorie, high-protein diet; drink at least 2 liters of water per day; take multivitamins as prescribed; maintain a symptom journal in relation to daily dietary intake; and avoid triggers

A 12-month-old client has a high blood lead level of 18 mcg/dL (0.87 µmol/L). The nurse educates the parents about lead poisoning. Which statements made by the parent indicate that teaching is successful? Select all that apply. 1. "I should get our home inspected for the source of the lead." 2. "I will vacuum our hard-surface floors daily." 3. "I will wash my child's hands often, especially before eating." 4. "We should use hot tap water for cooking." 5. "We will have to return for a follow-up lead level."

1. "I should get our home inspected for the source of the lead." 3. "I will wash my child's hands often, especially before eating." 5. "We will have to return for a follow-up lead level." Lead poisoning occurs from repeated lead exposure, either via ingestion of lead-based paints (eg, walls, toys), glazes (eg, pottery) or water from lead pipes, or by inhalation of contaminated dust or soil found around older homes. Elevated blood lead levels (BLLs) impair neural, blood, and renal development. A BLL screening is recommended between ages 1 and 2, or up to age 6 if the child was not previously screened. Clients with elevated BLLs (≥5 mcg/dL [0.24 µmol/L]) require follow-up blood work to ensure that levels decrease (Option 5). Chelation therapy may be required if levels remain elevated. The priority intervention for clients with elevated BLLs is preventing continued exposure. The home environment should be assessed for lead sources (Option 1). Pediatric and pregnant clients should not live in homes being renovated until the work is complete. Handwashing, especially before eating, is important to remove lead residue (Option 3). Educational objective:Pediatric clients are at risk for lead poisoning from environmental exposure in the home (eg, paint, dust, plumbing). Clients with elevated blood lead levels require monitoring. Lead sources in the home should be removed or mitigated (eg, handwashing, wet-dusting/mopping) to prevent further exposure.

The nurse is caring for a client who has a postoperative paralytic ileus following a bowel resection for colon cancer. The client is receiving patient-controlled analgesia (PCA) with morphine. Which nursing diagnoses (NDs) are appropriate to include in the client's care plan? Select all that apply. 1. Acute pain 2. Dysfunctional gastric motility 3. Imbalanced nutrition, less than body requirements 4. Ineffective self-health management 5. Risk for infection

1. Acute pain 2. Dysfunctional gastric motility 3. Imbalanced nutrition, less than body requirements 5. Risk for infection The following NDs are appropriate to include in this client's care plan: Acute pain related to tissue damage as evidenced by the use of PCA with an opioid. Pain is usually most intense 12-36 hours after surgery (Option 1). Dysfunctional gastric motility related to bowel manipulation during surgery, anesthesia, and opioid analgesia as evidenced by absent or hypoactive bowel sounds 48-72 hours following surgery secondary to a paralytic ileus. It is a common complication following abdominal surgery (Option 2). Imbalanced nutrition, less than body requirements related to the increased metabolic demand needed for tissue and wound healing as evidenced by the inability to ingest adequate caloric intake secondary to a paralytic ileus and the lack of interest in eating secondary to the ileus, the adverse effects of anesthesia, and analgesic medications (Option 3). Risk for infection - The risk for being invaded by pathogenic organisms is increased in this client due to loss of primary defenses (ie, protective skin barrier), lack of adequate nutrition to meet the body demands, and altered immunity due to the presence of cancer cells (Option 5). Educational objective:NDs associated with impaired skin integrity and wound healing following abdominal surgery that guide supportive and preventive nursing care include acute pain, dysfunctional gastric motility, imbalanced nutrition (less than body requirements), and risk for infection.

The nurse is caring for a client with acute diverticulitis who has nausea, vomiting, and rates pain as 8 on a scale of 0-10. Which of the following interventions should be included in the plan of care? Select all that apply. Vital signs Temperature 101.2 F (38.4 C) Blood pressure 126/70 mm Hg Heart rate 102/min Respirations 20/min SpO2 98% 1. Administer morphine sulfate as prescribed for pain control 2. Insert a rectal tube to protect the client's skin from diarrhea 3. Instruct the client to avoid straining 4. Maintain NPO status 5. Start IV infusion of normal saline

1. Administer morphine sulfate as prescribed for pain control 3. Instruct the client to avoid straining 4. Maintain NPO status 5. Start IV infusion of normal saline Diverticulosis is a condition in which saclike protrusions (ie, diverticula) develop in the large intestine, caused by increased intraabdominal pressure (eg, straining, lifting, tight clothing) and/or chronic constipation. When diverticula become infected and inflamed, the individual has diverticulitis. Acute care for diverticulitis focuses on allowing the colon to rest and the inflammation to resolve. This includes: NPO status: More acute cases require complete rest of the bowel; less severe cases may be handled at home, and clients may tolerate a low-fiber or clear liquid diet (Option 4) IV fluids to prevent dehydration when NPO (Option 5) Pain relief with IV medications to maintain NPO status: Opioids (eg, morphine sulfate) are indicated in moderate to severe cases of acute diverticulitis that require hospitalization (Option 1) Preventing increased intraabdominal pressure (eg, straining, coughing, lifting) to avoid perforation and rupture (Option 3) Educational objective:Management of acute diverticulitis focuses on bowel rest (NPO status, bed rest) and drug therapy (IV antibiotics, analgesics). Any procedure or treatment that increases intraabdominal pressure or may cause rupture of the inflamed diverticula should be avoided.

The nurse cares for a client with ulcerative colitis who is having abdominal pain and ≥10 bloody stools per day. Which of the following interventions should be included in the client's plan of care? Select all that apply. 1. Administer prescribed analgesic medications as needed 2. Encourage the client to discuss feelings about illness 3. Initiate strict, hourly intake and output monitoring 4. Investigate the client's compliance with the medication regimen 5. Offer the client high-protein foods during meals and snacks

1. Administer prescribed analgesic medications as needed 2. Encourage the client to discuss feelings about illness 3. Initiate strict, hourly intake and output monitoring 4. Investigate the client's compliance with the medication regimen 5. Offer the client high-protein foods during meals and snacks Ulcerative colitis (UC) is a chronic inflammatory bowel disease characterized by bouts of bloody diarrhea, abdominal pain, anorexia, and anemia. Nurses caring for clients with UC should provide pain management, promote adequate nutrition and hydration, address psychosocial needs, and evaluate client compliance with treatment.

The nurse is caring for a client who had an open appendectomy 24 hours ago. Based on the client's clinical data, which of the health care provider's new prescriptions should the nurse clarify? Exhibit Surgical wound is clean, dry & well approximated. Indwelling urinary catheter is draining clear, yellow urine. Diminished lung sounds auscultated in bilateral bases. Bowel sounds are absent in all four quadrants. Client denies the passage of flatus since surgery. 1. Advance client to regular diet 2. Ambulate 3 times daily postoperative day 1 3. Increase IV normal saline rate from 20 mL/hr to 100 mL/hr 4. Remove indwelling urinary catheter

1. Advance client to regular diet Postoperative ileus refers to the temporary deceleration of gastrointestinal motility following surgery, which can occur as a result of decreased PO intake, anesthesia, intestinal handling during surgery, and opioid use. Signs of postoperative ileus include abdominal distension, vomiting, and an inability to pass flatus or stool. Following abdominal surgery (eg, appendectomy), clients usually begin a clear liquid diet, and the diet is advanced gradually as gut motility returns to normal (eg, client tolerating intake, passage of flatus, bowel movement, return of bowel sounds). Advancing to a regular diet before the return of peristalsis could cause a bowel obstruction (Option 1). Educational objective:Following abdominal surgery, the nurse should not advance the client to a regular diet until peristalsis returns (eg, bowel sounds, passage of flatus, bowel movement). Advancing the diet before peristalsis returns could lead to a bowel obstruction.

A client admitted 3 days ago with upper gastrointestinal bleeding underwent an endoscopic procedure to stop the bleeding. The client is started on a clear liquid diet today. Which foods are appropriate for the nurse to offer the client? Select all that apply. 1. Apple juice 2. Cherry popsicle 3. Chicken broth 4. Frozen yogurt 5. Unsweetened tea 6. Vanilla ice cream

1. Apple juice 3. Chicken broth 5. Unsweetened tea Unsweetened tea, chicken broth, and apple juice are appropriate food choices for a client on a clear liquid diet. Popsicles are part of a clear liquid diet. However, red dyes in clear liquids (eg, cherry popsicles, red gelatin) should not be given to clients with recent gastrointestinal bleeding. A postoperative diet begins with ice chips and progresses to clear liquids, full liquids, soft diet, and then regular diet. Clear liquids with red dyes should not be given to clients with recent gastrointestinal bleeding.

The nurse is caring for a client in the postanesthesia care unit following a gastroduodenostomy. Which of the following nursing interventions are appropriate? Select all that apply. 1. Applying bilateral sequential compression devices 2. Encouraging splinting of the incision with a pillow when coughing 3. Keeping the client NPO until bowel sounds return 4. Maintaining supine positioning at all times 5. Repositioning and irrigating a clogged nasogastric tube PRN

1. Applying bilateral sequential compression devices 2. Encouraging splinting of the incision with a pillow when coughing 3. Keeping the client NPO until bowel sounds return Postoperative care of a client with gastroduodenostomy includes initiation of thromboembolism prophylaxis; turning, coughing, and deep breathing; and aspiration precautions (eg, elevating the head of the bed). The nurse should keep clients NPO until bowel sounds return and should not manipulate clogged nasogastric tubes.

During morning rounds, the nurse notices that a client admitted 3 days ago with hepatic encephalopathy is sleepy and confused. The client is scheduled for discharge later today. Which interventions are appropriate for the nurse to implement? Select all that apply. 1. Assess the client's hand movements with the arms extended 2. Compare current mental status findings with those from previous shifts 3. Contact the health care provider to request a blood draw for ammonia level 4. Encourage the client to ambulate in the hallway 5. Hold the client's morning dose of lactulose

1. Assess the client's hand movements with the arms extended 2. Compare current mental status findings with those from previous shifts 3. Contact the health care provider to request a blood draw for ammonia level Hepatic encephalopathy is a serious complication of end-stage liver disease (ESLD) that results from inadequate detoxification of ammonia from the blood. Symptoms include lethargy, confusion, and slurred speech; coma can occur if this condition remains untreated. Asterixis, or a flapping tremor of the hands when the arms are extended with the hands facing forward, may also be noted in the client with encephalopathy. The client with ESLD exhibiting confusion and lethargy should be evaluated for worsening encephalopathy by assessing for asterixis and comparing current mental status and ammonia level to previous findings. If encephalopathy continues to worsen, medical treatment should include higher doses of lactulose and rifaximin, and discharge should be delayed until the client is stable. Educational objective:Hepatic encephalopathy is a serious complication of end-stage liver disease caused by high levels of ammonia in the blood. Assessment findings include confusion, lethargy, and asterixis; coma and death can occur if this condition remains untreated. Pharmacologic treatments include lactulose and antibiotics (eg, rifaximin). The client with worsening encephalopathy is not stable enough for discharge.

The parent of a 21-day-old male infant reports that the infant is "throwing up a lot." Which assessments should the nurse make to help determine if pyloric stenosis is an issue? Select all that apply. 1. Assess the parent's feeding technique 2. Check for family history of gluten enteropathy 3. Check for history of physiological hyperbilirubinemia 4. Check if the vomiting is projectile 5. Compare current weight to birth weight

1. Assess the parent's feeding technique 4. Check if the vomiting is projectile 5. Compare current weight to birth weight In pyloric stenosis, there is gradual hypertrophy of the pylorus until symptom onset at age 3-5 weeks. It is common in first-born boys and the etiology is unclear. Pyloric stenosis presents with postprandial projectile vomiting (ejected up to 3 feet) followed by hunger (eg, "hungry vomiter"). This is clearly distinguished from the "wet burps" infants have due to a weak lower esophageal sphincter. The emesis is nonbilious as the obstruction is proximal to the bile duct. Infants have poor weight gain and are often dehydrated (eg, sunken fontanelle, decreased skin turgor, delayed capillary refill). The amount of milk consumed (particularly with bottle feedings) along with the mother's technique (mainly adequate burping) should be assessed to ensure there is no excessive air swallowing or overfeeding as an etiology. Pyloric stenosis is a hypertrophy of the pyloric sphincter that eventually causes complete obstruction. Classic signs include projectile nonbilious vomiting, an olive-shaped right upper quadrant mass, weight loss, dehydration, and/or electrolyte imbalance (metabolic alkalosis).

The nurse is reinforcing discharge teaching for the parents of a 1-year-old with a newly diagnosed cow's milk allergy. Which nutrients normally provided by milk should be obtained from other sources? Select all that apply. 1. Calcium 2. Fiber 3. Iron 4. Vitamin D 5. Vitamin K

1. Calcium 4. Vitamin D Calcium and vitamin D are nutrients in cow's milk that are essential for proper bone development in children and should be obtained from other sources for clients with a cow's milk allergy. Alternate sources of calcium include beans, dark greens, and calcium-fortified cereal and juices. Vitamin D is synthesized in the skin when exposed to sunlight and can be obtained in foods such as fish, egg yolks, and vitamin D-fortified foods.

The nurse is reviewing lifestyle and nutritional strategies to help reduce symptoms in a client with newly diagnosed gastroesophageal reflux disease. Which strategies should the nurse include? Select all that apply. 1. Choose foods that are low in fat 2. Do not consume any foods containing dairy 3. Eat three large meals a day and minimize snacking 4. Limit or eliminate the use of alcohol and tobacco 5. Try to avoid caffeine, chocolate, and peppermint

1. Choose foods that are low in fat 4. Limit or eliminate the use of alcohol and tobacco 5. Try to avoid caffeine, chocolate, and peppermint Educational objective:Lifestyle and dietary measures that help prevent or minimize symptoms of gastroesophageal reflux disease include avoiding dietary triggers such as alcohol, caffeine, chocolate, peppermint, and high-fat foods. Clients should consume small, frequent meals and discontinue the use of tobacco products.

A client with a history of diverticular disease is being discharged after an episode of acute diverticulitis. Which instructions should be included in the discharge teaching plan to reduce the risk of future episodes? Select all that apply. 1. Drink plenty of fluids 2. Exercise regularly 3. Follow a low-residue diet 4. Include whole grains, fruits, and vegetables in the diet 5. Increase intake of red meat

1. Drink plenty of fluids 2. Exercise regularly 4. Include whole grains, fruits, and vegetables in the diet Clients with diverticulosis should take measures to prevent constipation (eg, high-fiber diet, increased fluid intake, regular exercise), which may help prevent recurring episodes of acute diverticulitis.

The nurse will implement which nursing actions when caring for a client recently diagnosed with a hiatal hernia? Select all that apply. 1. Elevate the head of the hospital bed 2. Instruct the client to avoid tobacco and caffeine 3. Offer small, frequent, low-fat meals 4. Provide a girdle to reduce the hernia 5. Teach the client to avoid lifting or straining

1. Elevate the head of the hospital bed 2. Instruct the client to avoid tobacco and caffeine 3. Offer small, frequent, low-fat meals 5. Teach the client to avoid lifting or straining Conditions that increase intraabdominal pressure (eg, pregnancy, obesity, ascites, tumors, heavy lifting) and weaken the muscles of the diaphragm may allow a portion of the stomach to herniate through an opening in the diaphragm, causing a hiatal hernia. A sliding hernia occurs when a portion of the upper stomach squeezes through the hiatal opening in the diaphragm. A paraesophageal hernia (rolling hernia) occurs when the gastroesophageal junction remains in place but a portion of upper stomach folds up along the esophagus and forms a pocket. Paraesophageal hernias are a medical emergency. Although hiatal hernias may be asymptomatic, many clients experience signs and symptoms commonly associated with gastroesophageal reflux disease (GERD), including heartburn, dysphagia, and pain caused by increased intraabdominal pressure or supine positioning. Interventions to reduce herniation include the following: Diet modification—avoid high-fat foods and those that decrease lower esophageal sphincter pressure (eg, chocolate, peppermint, tomatoes, caffeine). Eat small, frequent meals, and decrease fluid intake during meals to prevent gastric distension. Avoid consumption of meals close to bedtime and nocturnal eating (Option 3). Lifestyle changes—smoking cessation, weight loss (Option 2). Avoid lifting or straining (Option 5). Elevate the head of the bed to approximately 30 degrees—this can be done at home using pillows or 4 - 6 inch blocks under the bed (Option 1). Educational objective:Hiatal hernias occur due to a weakening diaphragm and increased intraabdominal pressure. Nursing interventions to prevent hiatal hernias are similar to those used for gastroesophageal reflux disease (GERD), and they focus on decreasing intraabdominal pressure.

A client with a history of cirrhosis has a new prescription for lactulose 30 mL four times a day. What does the nurse explain to the client about this medication? 1. It will decrease intestinal absorption of ammonia. 2. It will facilitate diuresis of excess fluid 3. It will promote renal excretion of bilirubin 4. It will reduce portal pressure contributing to esophageal varices

1. It will decrease intestinal absorption of ammonia. Lactulose is a syruplike liquid that decreases intestinal ammonia absorption in clients with liver disease and hepatic encephalopathy. Hepatic encephalopathy occurs when the failing liver does not adequately detoxify ammonia in the body, leading to changes in mental status and death if not adequately and promptly treated. The lactulose dosing frequency should be adjusted to ensure 2-3 soft stools per day with no confusion or lethargy. Lactulose controls ammonia levels in hepatic encephalopathy by reducing intestinal absorption of ammonia with excretion in the stool.

A homeless man known to have chronic alcoholism and who has not eaten for 8 days is undergoing nutritional rehabilitation via oral and enteral feedings. Which of the following findings would indicate that the client is developing refeeding syndrome? 1. Phosphorus 2.0 mg/dL (0.65 mmol/L), potassium 2.9 mEq/L (2.9 mmol/L), magnesium 1.0 mEq/L (0.5 mmol/L) 2. Phosphorus 4.0 mg/dL (1.29 mmol/L), potassium 3.5 mEq/L (3.5 mmol/L), magnesium 2.0 mEq/L (1.0 mmol/L) 3. Random blood glucose 60 mg/dL (3.3 mmol/L), sodium 120 mEq/dL (120 mmol/L), calcium 7.0 mg/dL (1.75 mmol/L) 4. Random blood glucose 100 mg/dL (5.6 mmol/L), sodium 140 mEq/dL (140 mmol/L), calcium 10.0 mg/dL (2.50 mmol/L)

1. Phosphorus 2.0 mg/dL (0.65 mmol/L), potassium 2.9 mEq/L (2.9 mmol/L), magnesium 1.0 mEq/L (0.5 mmol/L) Refeeding syndrome is a serious complication of nutritional replenishment. It is marked by declines in serum phosphorus, potassium, and/or magnesium (mnemonic PPM). Clients can also develop fluid overload. Low-calorie feedings and a gradual increase in calories can prevent refeeding syndrome. Electrolytes should be monitored frequently.

The nurse is caring for a client with cirrhosis of the liver. Which blood test values would the nurse typically anticipate to be elevated when reviewing the client's morning laboratory results? Select all that apply. 1. Albumin 2. Ammonia 3. Bilirubin 4. Prothrombin time 5. Sodium

2. Ammonia 3. Bilirubin 4. Prothrombin time The chronic, progressive destruction characteristic of cirrhosis causes bilirubin, ammonia, and coagulation studies (PT/INR and aPTT) to become elevated. Hyponatremia and hypoalbuminemia are to be expected.

A student nurse asks why enteral (tube) feedings, rather than total parenteral nutrition (TPN), are being administered to a client with sepsis and respiratory failure. Which is the best response by the registered nurse? 1. "Enteral feedings have no complications." 2. "Enteral feedings maintain gut integrity and help prevent stress ulcers." 3. "Enteral feedings provide higher calorie content. 4. "Risk of hyperglycemia is lower with enteral feedings than with TPN."

2. "Enteral feedings maintain gut integrity and help prevent stress ulcers." Stress ulcers are a common complication in critically ill clients because the gastrointestinal tract is not a preferential organ. In the presence of hypoxemia, blood is shunted to the more vital organs, increasing the risk of stress ulcers. The early initiation of enteral feedings helps preserve the function of the gut mucosa, limits movement of bacteria (translocation) from the intestines into the bloodstream, and prevents stress ulcers. Enteral feedings are also associated with lower risk of infectious complications compared with TPN. However, the mortality is the same. Educational objective:The enteral route is preferred for feeding. Enteral feedings maintain the integrity of the gut, prevent stress ulcers, and help prevent the translocation of bacteria into the bloodstream.

The clinic nurse supervises a graduate nurse who is teaching the parents of a 2-year-old with acute diarrhea about home management. The nurse would need to intervene when the graduate nurse provides which instruction? 1. "Do not administer antidiarrheal medications to your child." 2. "Follow the bananas, rice, applesauce, and toast diet for the next few days." 3. "Record the number of wet diapers and return to the clinic if you notice a decrease." 4. "Use a skin barrier cream such as zinc oxide in the diaper area until diarrhea subsides."

2. "Follow the bananas, rice, applesauce, and toast diet for the next few days." During bouts of acute diarrhea and dehydration, treatment focuses on maintaining adequate fluid and electrolyte balance. The first-line treatment is oral rehydration therapy, using oral rehydration solutions (ORSs) to increase reabsorption of water and sodium. Even if the diarrhea is accompanied by vomiting, ORS should still be offered in small amounts at frequent intervals. Continuing the child's normal diet (solid foods) is encouraged as it shortens the duration and severity of the diarrhea. The BRAT (bananas, rice, applesauce, and toast) diet is not recommended as it does not provide sufficient protein or energy. (Option 1) Use of antidiarrheal medications is discouraged as these have little effect in controlling diarrhea and may actually be harmful by prolonging some bacterial infections and causing fatal paralytic ileus in children. (Option 3) Parents should be taught to monitor their child for signs of dehydration by checking the amount of fluid intake, number of wet diapers, presence of sunken eyes, and the condition of the mucous membranes. (Option 4) Protecting the perineal skin from breakdown during bouts of diarrhea can be accomplished by using skin barrier creams (eg, petrolatum or zinc oxide). Educational objective:When a child is experiencing acute diarrhea, the priority is to monitor for dehydration. Treatment is accomplished with oral rehydration solutions and early reintroduction of the child's normal diet (usual foods).

The registered nurse is teaching the parent of a 6-year-old about behavioral strategies for treating fecal incontinence due to functional constipation. Which statement by the parent indicates a need for further teaching? 1. "I will give my child a picture book to look at during toilet time." 2. "I will give my child a reward for each bowel movement while sitting on the toilet." 3. "I will keep a log of my child's bowel movements, laxative use, and episodes of soiling." 4. "I will schedule regular toilet sitting time for my child."

2. "I will give my child a reward for each bowel movement while sitting on the toilet." A reward system is one of the behavioral strategies used in the treatment of functional incontinence (due to constipation). The reward is given to encourage the child's involvement in the treatment to restore normal bowel function. Rewards are given for the child's effort and participation, not for having bowel movements while sitting on the toilet.

The nurse is reinforcing education with the parents of a 2-year-old child about diet choices to promote growth. The family observes a strict vegan diet. Which of the following statements by the nurse are appropriate? Select all that apply. 1. "Diets consisting of legumes as the only protein source are sufficient for growth." 2. "It is important to feed your child fortified breads and cereals to help with iron intake." 3. "Preparing meals with vegetables and fruits will ensure sufficient vitamin B12 intake." 4. "Try to pair foods high in iron with foods high in vitamin C to aid iron absorption." 5. "Your child may require calcium and vitamin D supplementation due to lack of dairy intake."

2. "It is important to feed your child fortified breads and cereals to help with iron intake." 4. "Try to pair foods high in iron with foods high in vitamin C to aid iron absorption." 5. "Your child may require calcium and vitamin D supplementation due to lack of dairy intake." With careful monitoring of nutritional intake, a vegan diet (ie, excluding all animal-derived products [eg, meat, dairy, eggs]) can be appropriate for clients in all age groups. Pediatric clients consuming a vegan diet are at increased risk for nutritional deficiencies (eg, protein, calories, calcium, vitamin D, iron, vitamin B12) due to rapid growth and development. Nurses educating clients about preventing nutritional deficiencies in vegan diets should include information about: Pediatric clients consuming a vegan diet are at risk for dietary deficiencies (eg, iron, protein, calories, vitamin B12, calcium, vitamin D). Parent education about supplementation and adequate food sources of these nutrients is necessary

A client tells the nurse of wanting to lose 20 lb (9 kg) in time for the client's daughter's wedding, which is 16 weeks away. How many calories (kcal) will the client have to eliminate from the diet each day to meet this goal? 1. 450 kcal/day 2. 625 kcal/day 3. 860 kcal/day 4. 1,000 kcal/day

2. 625 kcal/day A reduction or energy expenditure of 3,500 calories (kcal) will result in a weight loss of 1 lb (0.45 kg). To lose 20 lb (9 kg), the client needs to reduce intake by a total of 70,000 kcal (3500 kcal x 20 lb [9 kg] = 70,000 kcal). Over a period of 16 weeks, this would require a daily reduction of: 625 kcal (70,000 kcal / [16 weeks x 7 days] = 625 kcal/day) Adding an exercise regimen to the client's daily routine would facilitate additional weight loss and/or reduce the need for severe caloric restriction. Educational objective:A reduction or energy expenditure of 3500 calories (kcal) will result in a weight loss of 1 lb.

The nurse is performing a physical examination on a 10-year-old client with abdominal discomfort. Which actions would be appropriate during the examination? Select all that apply. 1. Ask the accompanying parent to rate and describe the client's pain 2. Ask the client to describe the chief symptom 3. Conduct a head-to-toe assessment in the same manner as an adult assessment 4. Explain the outcome of the examination to the parent without the child present 5. Honor the client's request to be examined without a parent present

2. Ask the client to describe the chief symptom 3. Conduct a head-to-toe assessment in the same manner as an adult assessment 5. Honor the client's request to be examined without a parent present Assessment based on the client's developmental age includes the following: Clients as young as 3 can tell and/or show the examiner where they hurt or how they feel in their own terms 10-year-olds are capable of understanding and assisting in their physical examination. In fact, school-age clients are usually quite interested in equipment and how it works. 10-year-olds are becoming modest and do not want a parent, especially of the opposite sex, in the room with them during a physical examination Educational objective:When performing a physical assessment on a child, it is imperative that the examiner understand the developmental age so that the child will be more comfortable and cooperative during the examination.

The unlicensed assistive personnel (UAP) assists a client with cirrhosis who underwent paracentesis 4 hours ago. The UAP reports to the nurse that the client was lightheaded and unsteady while ambulating to the chair. Which action should the nurse implement first? 1. Ask the UAP to take a set of vital signs 2. Assess the symptoms reported by the UAP 3. Hold the prescribed diuretic medications 4. Instruct the UAP to assist the client to bed

2. Assess the symptoms reported by the UAP Paracentesis is a procedure that involves removal of excess fluid from the peritoneal cavity (ascites) and is performed to relieve dyspnea and discomfort related to increased intra-abdominal pressure and fluid volume. Hypovolemia is an associated complication related to intravascular fluid shifts that occur during and post-procedure and also to high volume peritoneal fluid removal (>5 L). The nurse should first validate the presence of light-headedness and unsteady gait, monitor vital signs, and assess for manifestations of hypovolemia (eg, orthostatic hypotension, tachycardia, reduced pulse volume, decreased urine output), as decreased circulating volume can lead to hemodynamic instability. Educational objective:A client who is experiencing lightheadedness and unsteady gait following paracentesis requires immediate assessment because these manifestations can signal hypovolemia with hypotension, which can lead to hemodynamic instability and hypovolemic shock.

The nurse is caring for a client with a balloon tamponade tube in place due to bleeding esophageal varices. The client suddenly develops respiratory distress, and the nurse finds that the tube has been partially pulled out. Which intervention should be the nurse's priority? 1. Contact the health care provider 2. Cut the tube with scissors 3. Increase gastric suction level 4. Place the client in high Fowler position

2. Cut the tube with scissors Airway obstruction can occur if the balloon tamponade tube becomes displaced and a balloon migrates into the oropharynx. Scissors are kept at the bedside as a precaution; in the event of airway obstruction, the nurse can emergently cut the tube for rapid balloon deflation and tube removal. A balloon tamponade tube is used to compress bleeding esophageal varices. Tube displacement may result in airway obstruction. The nurse should keep scissors at the bedside so that the tube can be emergently cut and removed if respiratory distress develops due to tube displacement.

During the immediate postoperative period after a colostomy, which stoma appearance requires the nurse to contact the health care provider (HCP) immediately? 1. Brick red with slight moisture noted. 2. Dusky with moderate edema present. 3. Pink with slight oozing of blood. 4. Rosy with no stool produced

2. Dusky with moderate edema present. Educational objective:A healthy stoma has the characteristics of mucosal tissue and should appear vascular and moist. Indications of decreased blood supply (pale, dusky, or cyanotic) should be reported to the HCP immediately.

The nurse is counseling a client with obesity who is starting a weight reduction diet. The client reports consuming 4-5 regular cola beverages daily. Which of the following beverages should the nurse recommend as healthier substitutes? Select all that apply. 1. Commercial fruit juice 2. Flavored club soda 3. Fresh vegetable juice 4. Sports beverages 5. Unsweetened tea

2. Flavored club soda 3. Fresh vegetable juice 5. Unsweetened tea Substituting low- or zero-calorie beverages such as water, unsweetened tea, fresh vegetable juice, and club soda for high-calorie, sugary beverages can have a significant impact on weight loss for clients who have habitually consumed them.

The school nurse creates a cafeteria menu for a newly enrolled child with celiac disease. Which lunches would be appropriate for this child? Select all that apply. 1. Beef barley soup with mixed vegetables and French bread 2. Grilled chicken, baked potato, and strawberry yogurt 3. Mexican corn tacos with ground beef and cheese 4. Peanut butter and jelly on rice cakes with an oatmeal cookie 5. Rice noodles with chicken and broccoli

2. Grilled chicken, baked potato, and strawberry yogurt 3. Mexican corn tacos with ground beef and cheese 5. Rice noodles with chicken and broccoli Celiac disease (celiac sprue) is an autoimmune disorder in which the body is unable to process gluten, a protein found in most grains. Gluten consumption will damage the villi of the small intestine; this results in malabsorption of fats (steatorrhea, foul-smelling stools) and other nutrients, which can lead to malnutrition and failure to thrive. The child will need to adhere to a gluten-free diet for life. Rice, corn, and potatoes are gluten free and are allowed in the diet (Options 2, 3, and 5). A child with celiac disease cannot eat barley, rye, oats, or wheat (mnemonic - BROW). Educational objective:Celiac disease is an autoimmune disorder in which an individual cannot tolerate gluten, a protein found in barley, rye, oats, and wheat (BROW). Rice, corn, and potatoes are allowed in the diet and can be used as grain substitutes. Affected individuals must adhere to a gluten-free diet for life.

A client is receiving an infusion of total parenteral nutrition (TPN) with 20% dextrose through a central line at 75 mL/hr. The nurse responds to the client's IV pump alarm, which indicates that the bag is empty. The new bag is not expected to arrive from the pharmacy for an hour. What is the most appropriate nursing action? 1. Hang 0.9% normal saline until new bag arrives, then increase TPN to 150 mL/hr for 1 hour 2. Hang 10% dextrose in water until the new bag arrives, then resume TPN at 75 mL/hr 3. Hang dextran in saline until the new bag arrives, then resume TPN at 75 mL/hr 4. Hang lactated Ringer's until the new bag arrives, then resume TPN at 75 mL/hr

2. Hang 10% dextrose in water until the new bag arrives, then resume TPN at 75 mL/hr otal parenteral nutrition (TPN) is administered via a central venous catheter to meet the nutritional needs (eg, glucose, amino acids, vitamins, minerals) of clients who cannot digest nutrients via the gastrointestinal tract. The nurse should hang 10% dextrose in water at the same infusion rate of 75 mL/hr until the new bag arrives. If the 20% dextrose solution is temporarily replaced with an infusion lacking dextrose (eg, normal saline, lactated Ringer's [LR]), the pancreas will continue to produce insulin in response to the residual glucose, which may cause hypoglycemia. Abrupt cessation of central total parenteral nutrition (TPN), which usually contains 20%-50% dextrose, increases the risk for hypoglycemia, as the pancreas will continue to produce insulin in response to the residual glucose. When TPN is discontinued, the infusion rate is gradually reduced and then replaced with a solution containing dextrose.

The nurse is gathering data on a 5-week-old admitted with a suspected diagnosis of pyloric stenosis. The nurse should expect to find which laboratory value? 1. Blood pH of 7.1 2. Hematocrit of 57% (0.57) 3. Potassium of 5.2 mEq/L (5.2 mmol/L) 4. White blood cells of 28,500/mm3 (28.5 x 109/L)

2. Hematocrit of 57% (0.57) In pyloric stenosis, a hypertrophied pyloric muscle causes postprandial projectile vomiting secondary to an obstruction at the gastric outlet. An olive-shaped mass may be palpated in the epigastric area just to the right of the umbilicus. Emesis is nonbilious (formula in/formula out) and leads to progressive dehydration. Infants will be hungry constantly despite regular feedings. A hematocrit of 57% (0.57) is elevated and indicative of hemoconcentration caused by dehydration. Elevated blood urea nitrogen is also a sign of dehydration. Hypertrophic pyloric stenosis results in recurrent projectile vomiting, which leads to dehydration and hypokalemic metabolic alkalosis. Dehydration is manifested by hemoconcentration (elevated hematocrit) and elevated blood urea nitrogen.

The nurse assesses a client with suspected acute pancreatitis and anticipates the client reporting pain in which anatomical area? 1. Left flank radiating to the left groin area 2. Left upper quadrant radiating to the back 3. Periumbilical area shifting to the right lower quadran 4. Right upper quadrant radiating to the right shoulder

2. Left upper quadrant radiating to the back The client with acute pancreatitis has sudden, severe pain in the left upper quadrant or midepigastric area of the abdomen, often radiating to the back. Clients are at risk of developing hypovolemia (third spacing of fluids), acute respiratory distress syndrome (due to intense systemic inflammatory response), and hypocalcemia (necrosed fat binding calcium).

The nurse is caring for a client with cirrhosis. Assessment findings include ascites, peripheral edema, shortness of breath, fatigue, and generalized discomfort. Which interventions would be appropriate for the nurse to implement to promote the client's comfort? Select all that apply. 1. Encourage adequate sodium intake 2. Place client in semi-Fowler position 3. Place client in Trendelenburg position 4. Provide alternating air pressure mattress 5. Use music to provide a distraction

2. Place client in semi-Fowler position 4. Provide alternating air pressure mattress 5. Use music to provide a distraction In a client with cirrhosis and ascites, discomfort is often due to pressure of the fluid on the surrounding organs. Shortness of breath occurs due to the upward pressure exerted by the abdominal ascites on the diaphragm, which restricts lung expansion. Positioning the client in semi-Fowler or Fowler position can promote comfort, as this position can reduce the pressure on the diaphragm (Option 2). In semi-Fowler position, the head of the bed is elevated 30-45 degrees; in Fowler position, elevation is 45-60 degrees. Side-lying with the head elevated can also be a position of comfort for the client with ascites as it allows the heavy, enlarged abdomen to rest on the bed, reducing pressure on internal organs and allowing for relaxation. Meticulous skin care is a priority due to the increased susceptibility of skin breakdown from edema, ascites, and pruritus. It is important to use a specialty mattress and implement a turning schedule of every 2 hours (Option 4). A distraction can take the client's mind off the current symptoms and may also help promote comfort in many different situations. Some of these distractions include listening to music, watching television, playing video games, or taking part in hobbies (Option 5). Educational objective:The client with discomfort and shortness of breath due to ascites should be positioned in the semi-Fowler or Fowler position to promote comfort and lung expansion. Music and other methods of distraction may also promote comfort. Meticulous skin interventions (eg, specialty mattress, turning schedule) are important to prevent tissue breakdown.

The nurse is assessing a 3-month-old infant who was admitted to the floor 18 hours ago after undergoing surgical repair of a cleft lip. Which assessment finding would cause the nurse to be concerned? 1. The area around the incision appears swollen 2. The client is prone while playing with the parent 3. The client is wearing bilateral elbow restraints 4. There are adhesive strips over the incision

2. The client is prone while playing with the parent Cleft lip and cleft palate are common congenital defects that can occur separately or together. Cleft lip is an opening (ie, notch) in the upper lip, and cleft palate is a malformation of the roof of the mouth due to incomplete fusion of the palatine bones and maxilla during fetal development. Surgery to repair a cleft lip and/or palate is usually performed between age 2 months to 12 months. Postoperative care includes protecting the surgical site from trauma to facilitate proper healing. The nurse should ensure the infant is positioned to facilitate secretion drainage and prevent airway obstruction (eg, upright, supine). The nurse should avoid placing the infant prone or on the side of the incision because of the risk for damage to the surgical site due to bumping or rubbing the face against the mattress or floor (Option 2). Educational objective:Cleft lip and cleft palate are common congenital defects that are usually surgically repaired during infancy. Postoperative care includes protecting the surgical site from trauma. The infant should not be in a prone position because of the risk for damage to the surgical site due to rubbing or bumping it against the mattress and sheets.

The clinic nurse is interviewing the parents of a 6-month-old client about the infant's diet. Which statement by the parents is most concerning? 1. "Because apples are healthy, we make apple pie and feed small, soft bites to our baby." 2. "If our baby refuses to finish foods, we continue to offer small bites, so food isn't wasted." 3. "Infant oatmeal sweetened with fresh honey is our baby's favorite breakfast." 4. "We found that the food in TV dinners can be easily pureed and is convenient."

3. "Infant oatmeal sweetened with fresh honey is our baby's favorite breakfast." Educational objective:Clostridium botulinum spores in honey can colonize an infant's (age <12 months) immature gastrointestinal system and release a toxin that causes botulism, a rare but potentially life-threatening illness.

The mother of a 6-year-old child with cystic fibrosis (CF) has received instruction on the use of pancreatic enzymes. Which statement made by the mother indicates a need for further teaching? 1. "I need to monitor the total amount of this medication that I give to my child every day." 2. "I should give this medication with or just before my child has a meal or snack." 3. "It is okay for my child to chew this medication." 4. "It is okay to open the capsule and sprinkle the medicine on a tablespoon of applesauce."

3. "It is okay for my child to chew this medication." In CF, unusually thick mucus obstructs the pancreatic ducts, preventing pancreatic enzymes (amylase, trypsin, and lipase) from reaching the small intestine. The result is malabsorption of carbohydrates, fats, and proteins; the inability to absorb fat-soluble vitamins (A, D, E, and K) is of particular concern. Gastrointestinal signs and symptoms of CF include flatulence, abdominal cramping, ongoing diarrhea, and/or steatorrhea. Nutritional therapy includes the administration pancreatic enzyme supplements with or just before every meal or snack (Option 2). Educational objective:Pancreatic enzyme supplements are used to aid the absorption of carbohydrates, fats, and proteins in a child with CF. They are taken with or just before every meal (not as needed); should be swallowed whole or sprinkled on an acidic food; and should not be crushed or chewed. They should not be taken with milk. Excessive intake could result in fibrosing colonopathy.

Which of the following statements made by the mother of a child recently diagnosed with celiac disease indicates a need for further teaching? 1. "I will need to read the labels of all processed foods." 2. "It is okay if my child eats rice, corn, and potatoes." 3. "My child can have small amounts of foods containing wheat as long as she remains symptom free." 4. "My child will need to be on a gluten-free diet for the rest of her life."

3. "My child can have small amounts of foods containing wheat as long as she remains symptom free." All sources of gluten must be eliminated from the diet of a client with celiac disease; consuming small amounts, even in the absence of clinical symptoms, will increase the risk of damage to the intestinal villi. Clients can have foods containing rice, corn, and potatoes. They should read food labels and follow the diet for the rest of their lives.

The nurse is discussing feeding and eating practices with the mother of a 1-year-old. Which statement made by the mother indicates a need for further instruction? 1. "I give my child chopped fruit rather than juice." 2. "I make sure my child drinks plenty of water between meals." 3. "My child is fussy at bedtime so I put him to sleep with a bottle of milk." 4. "When I give my child a new food, I wait a week before trying a second new food."

3. "My child is fussy at bedtime so I put him to sleep with a bottle of milk." The practice of using a bottle with milk to calm a child at bedtime should be discouraged. A child who falls asleep drinking a fluid high in carbohydrates will likely develop extensive dental caries, a condition known as baby bottle tooth decay

An adult diagnosed with celiac disease 3 weeks ago was placed on a gluten-free diet. The client returns for ambulatory care follow-up, reports continuation of symptoms, and does not seem to be responding to therapy. Which is the best response by the nurse? 1. "I will refer you to the dietitian." 2. "It should take about 6-8 weeks before you see improvement in your symptoms." 3. "Tell me what you had to eat yesterday." 4. "You must not be following your diet.

3. "Tell me what you had to eat yesterday." This client with celiac disease continues to have symptoms. An assessment of the client's food intake must be obtained to determine if it includes foods that contain gluten, a protein in barley, rye, oats, and wheat (mnemonic: BROW). The most common reason for non-responsiveness to a gluten-free diet in clients with celiac disease is that gluten has not been entirely eliminated from their food intake. When a client with celiac disease does not experience symptom relief after being on a gluten-free diet, it is most important for the nurse to assess the underlying cause. The most common reason for refractory symptoms is failure to follow the strict gluten-free diet.

The clinic nurse is caring for several clients during well-child visits. The nurse should recognize which client as being the most at risk for anemia? 1. 1-month-old infant born at term gestation who exclusively breastfeeds( 2. 2-month-old infant born at preterm gestation who exclusively receives iron-fortified formula 3. 3-month-old infant born at preterm gestation who is exclusively bottle-fed with breastmilk 4. 6-month-old infant born at term gestation who breastfeeds and eats iron-fortified infant cereal

3. 3-month-old infant born at preterm gestation who is exclusively bottle-fed with breastmilk Iron deficiency during infancy causes reduced hemoglobin production, resulting in anemia, decreased immune function, and delayed growth and development. During gestation, the fetus stores iron received from the mother; the amount of iron stored is dependent on the length of gestation. After birth, iron stores are progressively depleted and nutritional sources of iron are eventually required. Infants born at preterm gestation have less time in utero to accumulate iron. Preterm infants typically deplete iron stores by age 2-3 months and require additional iron supplementation (eg, oral iron drops, iron-fortified formula). Therefore, a 3-month-old infant born at preterm gestation who is exclusively receiving breastmilk is most at risk for anemia. During gestation, the amount of iron a fetus stores is dependent on the length of gestation. Infants born at preterm gestation have lower iron stores at birth and are at an increased risk for iron-deficiency anemia. Iron supplementation (eg, oral iron drops, iron-fortified formula) is usually needed by preterm infants at an earlier age (2-3 months).

The parent of a 7-month-old reports that the child has been crying and vomiting with a distended belly for the past 4 hours. The infant is now lying quietly in the parent's arms with a pulse of 200/min and respirations of 60/min. Which of the following components of SBAR (situation, background, assessment, recommendation/read-back) communication is most important for the nurse to report to the health care provider? 1. Client has been ill for approximately 4 hours 2. Client has improved from apparent earlier distress 3. Client is now lethargic with abnormal vital signs 4. Does the health care provider want to order a laxative?

3. Client is now lethargic with abnormal vital signs SBAR (situation, background, assessment, recommendation/read-back) is an established reporting format used to communicate with the health care provider (HCP). Use of SBAR ensures that the HCP receives the necessary information to make a clinical judgment regarding treatment or need for immediate assessment. In this situation, the client's presentation indicates worsening symptoms that require immediate intervention. The client's lethargy represents a declining level of consciousness. The client also has significantly abnormal vital signs (normal infant pulse rate is 110-160/min, respirations generally around 40/min). These are ominous signs that should be reported immediately

During a routine assessment of a developmentally normal 18-month-old, the parent expresses concern about the small amount of food the child consumes. What is the nurse's priority intervention? 1. Check the child for parasitic infections 2. Consult a pediatric nutritionist for suspected eating disorder 3. Educate the parent about physiologic anorexia 4. Notify the primary health care provider

3. Educate the parent about physiologic anorexia Physiologic anorexia occurs when the very high metabolic demands of infancy slow down to keep pace with the moderate growth of toddlerhood. During this phase, toddlers are increasingly picky about their food choices and schedules. Although to the parents it may appear that the child is not consuming enough calories, intake over several days actually meets nutritional and energy needs. Parents should be educated concerning what constitutes a healthy diet for toddlers and which foods they are more likely to consume. Some strategies for dealing with a toddler during a stage of physiologic anorexia and pickiness include: Set and enforce a schedule for all meals and snacks Offer the child 2 or 3 choices of food items Do not force the child to eat Keep food portions small Expose the child repeatedly to new foods on several separate occasions Avoid TV and games during meals or snacks Physiologic anorexia is a normal period of decreased appetite that occurs in toddlers around age 18 months as a result of decreased metabolic needs. Parents should be taught to provide multiple food options, set a schedule for meals/snacks, and avoid watching TV or playing games during meal time; toddlers should not be forced to eat.

The nurse teaching a group of clients about celiac disease will include which meal in the teaching plan? 1. Baked salmon with rice, steamed vegetables, and dinner roll 2. Breaded pork chops, corn on the cob, and steamed snow peas 3. Grilled chicken, green beans, and mashed potatoes 4. Spaghetti with Italian tomato sauce and meatballs

3. Grilled chicken, green beans, and mashed potatoes Celiac disease is an autoimmune disorder in which chronic inflammation caused by gluten damages the small intestine. The following are important dietary principles to teach clients with celiac disease: All gluten-containing products should be eliminated from the diet. These include wheat, barley, rye, and oats. Rice, corn, and potatoes are gluten free and are allowed on the diet. All sources of gluten must be eliminated from the diet of a client with celiac disease. Consuming small amounts, even in the absence of clinical symptoms, will increase the risk for damage to the intestinal villi. Clients can have foods containing rice, corn, and potatoes. They should read food labels and follow the diet for the rest of their lives.

When assessing a client with cholelithiasis and acute cholecystitis, which findings might the nurse note during the health history and physical examination? Select all that apply. 1. Flank pain radiating to the groin 2. High-protein food ingestion before the onset of pain 3. Low-grade fever with chills 4. Pain at the umbilicus 5. Right upper-quadrant (RUQ) pain radiating to the right shoulder

3. Low-grade fever with chills 5. Right upper-quadrant (RUQ) pain radiating to the right shoulder Flank pain radiating to the groin is seen with renal colic (ureteral stones). Initial onset of pain at the umbilicus is seen with acute appendicitis. Education : Cardinal symptoms of acute calculous cholecystitis include pain in the RUQ and referred pain to the right shoulder and scapula a few hours after eating fatty foods. Associated symptoms include fever, chills, nausea, vomiting, and anorexia.

A client calls the primary care clinic reporting diarrhea for 4 days and a low grade fever. What instruction is most important for the nurse to give to the client? 1. Encourage client to eat bulk-forming foods such as whole grain bread. 2. Encourage rest, fluids, and acetaminophen for the fever 3. Make an appointment for the client with the health care provider today. 4. Take 2 tablets of loperamide followed by 1 tablet after each loose stool.

3. Make an appointment for the client with the health care provider today. Clients experiencing diarrhea lasting >48 hours or accompanied by fever or bloody stool should see their health care provider for assessment of fluid status, electrolyte levels, and identification of underlying causes.

The nurse is caring for a client who underwent an endoscopic sleeve gastrectomy this morning for the treatment of morbid obesity. Based on the client's clinical data, which action by the nurse is appropriate at this time? Click the exhibit button for more information. 1. Assist the client to ambulate in the hall to dispel retained carbon dioxide 2. Give a PRN bolus of fentanyl from the client's PCA pump 3. Notify the health care provider or surgeon immediately 4. Reposition the client's nasogastric tube to ensure patency

3. Notify the health care provider or surgeon immediately Clients with morbid obesity who are unable to achieve weight reduction with conservative treatment (eg, lifestyle modification, medications) may choose to undergo bariatric surgery. Weight reduction from bariatric surgery occurs by restricting the stomach's capacity to hold food and/or reducing food absorption. During sleeve gastrectomy, most of the stomach is removed, and the remaining portions are attached together, creating a "sleeve" or pouch. After bariatric surgery, the nurse must closely monitor for clinical indicators of complications. Severe abdominal pain (especially radiating to the back/shoulder) accompanied by signs of sepsis (eg, restlessness, tachycardia, oliguria) may indicate an anastomotic leak, which occurs when gastric and/or intestinal contents leak through a surgical connection into the intrabdominal space. Clients with signs of anastomotic leaking should be reported to the health care provider because rapid development of septic shock and death are likely without immediate intervention (Option 3). Educational objective:After bariatric surgery, the nurse should monitor the client for severe abdominal pain (especially radiating to the back/shoulders), tachycardia, restlessness, and oliguria. These findings suggest a life-threatening anastomotic leak that the nurse should immediately report to the health care provider.

A client with ascites due to cirrhosis has increasing shortness of breath and abdominal pain. The health care provider requests that the nurse prepare the client for a paracentesis. Which nursing actions would the nurse implement prior to the procedure? Select all that apply. 1. Educate client about the procedure and obtain informed consent 2. Initiate NPO status 6 hours prior to the procedure 3. Obtain baseline vital signs, abdominal circumference, and weight 4. Place client in high Fowler position or as upright as possible 5. Request that the client empty the bladder

3. Obtain baseline vital signs, abdominal circumference, and weight 4. Place client in high Fowler position or as upright as possible 5. Request that the client empty the bladder Verify that the client received necessary information to give consent and witness informed consent. Instruct the client to void to prevent puncturing the bladder. Assess the client's abdominal girth, weight, and vital signs. Place the client in the high Fowler position or as upright as possible. Paracentesis is an invasive procedure requiring delivery of informed consent by the health care provider (HCP). The HCP explains the benefits and risks of the procedure. The nurse's role is to witness informed consent and verify that it has occurred. NPO status is not required for paracentesis, which is often performed at the bedside or in an HCP's office using only a local anesthetic.

The nurse is caring for a client with end-stage liver failure from hepatitis C who is being seen in the clinic for worsening ascites. The client is treated in the infusion center with intravenous (IV) albumin, IV furosemide, and oral spironolactone. The following day the nurse checks the client's labs. Which of the following lab findings is most important for the nurse to communicate to the health care provider? 1. Albumin 2.5 g/dL (25 g/L) 2. INR 1.4 3. Potassium 3.0 mEq/L (3.0 mmol/L) 4. Sodium 131 mEq/L (131 mmol/L)

3. Potassium 3.0 mEq/L (3.0 mmol/L) Lab abnormalities common in liver failure include low albumin, elevated INR, and elevated liver function tests. A low serum potassium can increase the risk of hepatic encephalopathy and should be reported to the health care provider. Elevated serum ammonia confirms the hepatic encephalopathy diagnosis.

A client with a 10-year history of unipolar major depression has relapsed and is now hospitalized. The client is currently on phenelzine and weighs 115 lb (52.1 kg) but weighed 150 lb (68 kg) 3 months prior to admission. Which foods would be the best for this client? 1. Crackers and cheddar cheese 2. Hard-boiled egg with tomatoes 3. Steamed fish and potatoes 4. Tortilla chips with avocado dip

3. Steamed fish and potatoes Clients with unipolar major depression are likely to have reduced appetite and unintentional weight loss. Interventions to promote adequate nutritional intake include providing small frequent meals and snacks that are dense in protein and calories. In addition, drug-nutrient interactions need to be considered when choosing foods for a client on an MAOI.

The nurse assesses a client who has followed a vegan diet for several years. Which client statement indicates a potential nutritional deficiency? 1. "I have had some visual disturbances while driving at night." 2. "I have had trouble falling asleep over the past few months." 3. "Scaly patches of skin are developing on my elbows and knees." 4. "Sometimes my hands and feet get a tingling sensation."

4. "Sometimes my hands and feet get a tingling sensation." Clients following a vegan diet should be educated about vitamin B12 deficiency and the importance of supplementation and eating B12-fortified foods. Chronic vitamin B12 deficiency may precipitate megaloblastic anemia and neurological symptoms (eg, peripheral neuropathy, neuromotor impairment, memory loss).

An adolescent client seen in the ambulatory care center is going on a one-week fasting regimen of water and juice to jump start weight loss. The nurse's response is based on an understanding of which of the following? 1. Fasting for 7 days is not likely to cause health problems 2. Fasting spares protein in favor of fat metabolism 3. Fasting will help control hunger pangs in the long term 4. Initial weight loss during fasting is primarily from fluid loss

4. Initial weight loss during fasting is primarily from fluid loss Fasting can cause multiple health problems, including increased stress, slowing of the body's metabolism, muscle damage, fluid loss, increased hunger, depletion of nutrients, and physical symptoms such as headache, dizziness, fatigue, and muscle weakness.

The nurse is admitting a client with cholelithiasis and acute cholecystitis. Suddenly, the client vomits 250 mL of greenish-yellow stomach contents and reports severe pain in the right upper quadrant with radiation to the right shoulder. Which intervention would have the highest priority? 1. Administer promethazine 25 mg suppository 2. Infuse normal saline 100 mL/hour 3. Insert nasogastric tube to low suction 4. Maintain nothing-by-mouth (NPO) status

4. Maintain nothing-by-mouth (NPO) status The highest priority intervention for an actively vomiting client with acute cholecystitis is maintenance of strict NPO status to avoid additional stimulation of the gallbladder. Additional priorities include management of nausea and vomiting, pain, fluid balance, and gastric decompression.

Which prescription should the nurse question when caring for a hospitalized client diagnosed with acute diverticulitis? 1. Metronidazole 500 mg IV every 8 hours 2. Nasogastric (NG) tube to suction 3. Nothing by mouth (NPO) 4. Prepare for barium enema in AM

4. Prepare for barium enema in AM Management of acute diverticulitis focuses on bowel rest (NPO status, NG suction, bed rest), and drug therapy (IV antibiotics, analgesics). Any procedure or treatment that increases intraabdominal pressure or may cause rupture of the inflamed diverticula should be avoided.

A newborn had a bowel resection with temporary colostomy for Hirschsprung's disease. The nurse should alert the health care provider (HCP) for which assessment finding postoperatively? 1. Moderate amount of blood-tinged mucus from the stoma on postoperative day 2 2. Small amount of non-formed stool in the colostomy bag on postoperative day 6 3. Stoma bleeds a small amount during colostomy bag change on postoperative day 3 4. Stoma is gray-tinged at the edges but pink at the center on postoperative day 5

4. Stoma is gray-tinged at the edges but pink at the center on postoperative day 5 The colostomy stoma should be beefy red in the immediate postoperative period. Any discoloration to the stoma could indicate decreased blood supply to the area; the nurse should notify the HCP.


Conjuntos de estudio relacionados

AP GOV! Chapter One True and False (Can't Use Scatter :/)

View Set

soc 101 chapter 12--gender, sex, and sexuality

View Set

Clinical Research Coordinator Exam

View Set

Property and Casualty terminology chapter 2, AD Banker Ch. 2 - Property Ins, AD Banker Ch. 1 Producer: Property and Casualty Insurance, comp exam questions, property and casualty chapter 15 questions, property and casualty chapter 14 questions, prope...

View Set

Ch 6 & 7 Qs Wrong- NJ Laws, Rules, and Regulations

View Set

Medical Terminology - Chapter 14 - Special Topics

View Set

PSYC3305 Exam 3: Chapters 9 & 10

View Set

Terrorism and Homeland Security Chapter 8,11,12,13 & 15

View Set

The Child with Cardiovascular Dysfunction

View Set